You are on page 1of 114

Issue No.

1 Page 65 of 262
S E
MT
C
ST. THERESE- MTC COLLEGES SEAM 2
Iloilo, Philippines
E

Trim, Stability, Stress


R

C
E

OL
. T H

STUDENT LEARNING MODULE


LEGES
ST

Revision No. 2 Effectivity date: Reviewed by: Approved by:

01 September 2021 QMR President

Lesson 7

Learning Module 7.1 Movement of the center of gravity (5


hours)

Competence, Course Outcomes, and Learning Outcomes

Competence:

A-II/1 F3.C2: Maintain seaworthiness of the ship

Course Outcome/s:

Calculate ship stability in compliance with the IMO intact stability criteria under all
conditions of loading

Learning Outcomes:

1. Describe the effect on the center of gravity of the ship when the load is moved
within, added to, or removed from the ship.

Overview

When loading, discharging, or shifting cargo on a


ship, the center of gravity,' or condition for
equilibrium, changes. As cargo is moved, the
locations of masses aboard the ship change, and the
distance of those masses from the keel also change.
All of these changes affect the moments acting on a
vessel.

© All Rights Reserved.


Issue No. 1 Page 66 of 262
S E
MT
C
ST. THERESE- MTC COLLEGES SEAM 2
Iloilo, Philippines
E

Trim, Stability, Stress


R

C
E

OL
. T H

STUDENT LEARNING MODULE


LEGES
ST

Revision No. 2 Effectivity date: Reviewed by: Approved by:

01 September 2021 QMR President

In this lesson, we will learn how to calculate the new center of gravity for a vessel when her
cargo is loaded, discharged, or shifted. At the last part of this lesson the students have to
perform the check point provided.

Discussion

1. Key Concept

MOVEMENT OF THE CENTER OF GRAVITY


Effect of loading, discharging, and shifting
cargo

The center of gravity of a body is the point at which the body's entire weight may be
considered as concentrated such that, if supported at that point, the body would remain in
equilibrium. A simple example of the center of gravity in the middle of a seesaw. Knowing
where the center of gravity (G) of a ship is located is very important because of its effect on
its stability.

The center of gravity is measured from the keel of the ship. You will often be required to
find the new position of 'G' when cargo is moved. KG denotes the distance from the center
of gravity (G) to the keel (K). If G1 represents the new center of gravity, then GG1 denotes
the shift in the center of gravity, and KG1 represents the distance between the new center of
gravity and the keel. Also, if 'g' represents the center of gravity of cargo loaded, discharged,
or shifted, then 'Kg' represents the distance from the cargo to the keel.

The following formulae are used to calculate the shift (GGv) and new position (KG) of the
center of gravity due to:

1. GGV is the shift of G: a vertical component; GGH a horizontal


component

Loading new cargo moves the center of gravity towards the


added cargo

The formula calculates the vertical component of the shift of G:


GGv = w x d / W + w
© All Rights Reserved.
Issue No. 1 Page 67 of 262
S E
MT
C
ST. THERESE- MTC COLLEGES SEAM 2
Iloilo, Philippines
E

Trim, Stability, Stress


R

C
E

OL
. T H

STUDENT LEARNING MODULE


LEGES
ST

Revision No. 2 Effectivity date: Reviewed by: Approved by:

01 September 2021 QMR President


W = is the ship's initial displacement (mass)
w = is the weight of cargo loaded
d = is the vertical distance between G of the ship and the g of the loaded weight

Example 1:
A ship displaces 12 500 t and has an initial KG of 6.5 m.
Calculate the final KG if 1000 t of cargo is loaded into the
lower hold at Kg 3.0 m.

Solution:
GGv = w x d = 1000 x (6.5 – 3.0) = 0.259 m.
W+w 12500 + 1000

Initial KG 6.500 m
GGv down 0.259 m
Final KG 6. 241 m

Example 2:
A ship displaces 17200 t and has an initial KG of 8.4 m. Calculate the final GK if 1400 t cargo
is loaded onto the main deck at Kg 10.5 m.

Solution:
GGv = w x d = 1400 x (10.5 – 8.4) = 0.158 m
W+w 17200 + 1400

Initial KG 8.400 m
GGv up 0. 158 m
Final KG 8. 558 m

2. Discharging cargo moves the center of


gravity away from the location of the removed cargo.

GGv = w x d / W−w where


W = ship's initial displacement
w = weight of the cargo discharged
d = vertical distance between G of the ship and g of the
discharged weight

Example 1:
A ship has a displacement of 13400 t and an initial KG of
4.22 m. 320 t of deck cargo is discharged from a position Kg 7.14 m. Calculate the final KG
of the Ship.

© All Rights Reserved.


Issue No. 1 Page 68 of 262
S E
MT
C
ST. THERESE- MTC COLLEGES SEAM 2
Iloilo, Philippines
E

Trim, Stability, Stress


R

C
E

OL
. T H

STUDENT LEARNING MODULE


LEGES
ST

Revision No. 2 Effectivity date: Reviewed by: Approved by:

01 September 2021 QMR President

Solution:
GGv = w x d = 320 x (7.14 – 4.22) = 0.071 m down
W–w 13400 – 320

Initial KG 4.220 m
GGv 0.071 m
Final KG 4.149 m

Example 2:
A ship displaces 18000 t and has an initial KG of 5.30 m. Calculate the final KG if 10000 t of
cargo is discharged from the lower hold Kg 3.0 m.

Solution:
GGv = w x d = 10000 x (5.3 – 3.0) = 2.875 m
W–w 18000 – 10000

Initial KG 5.300 m
GGv up 2.875 m
Final KG 8.175 m

3. Shifting cargo moves the center of gravity in the same direction as the cargo
GGv = w x d / W
W = ship's displacement, including the weight being shifted
w = weight shifted
d = the distance through which the weight is shifted

In this instant, GGv is a shift of G upward, i.e., KG increases

Example 1:
A ship displaces 5000 t and has an initial KG of 4.5 m.
Calculate the final KG if a weight of 20 t is moved vertically
upwards from the lower hold of Kg 2 m to the upper deck
of Kg 6.5 m.

Solution:
GGv = w x d = 20 x (6.5 – 2.0) = 20 x 4.5 = 0.018 m
W 5000 5000

Initial KG 4.500 m
GGv 0.018 m
Final KG 4.518 m

© All Rights Reserved.


Issue No. 1 Page 69 of 262
S E
MT
C
ST. THERESE- MTC COLLEGES SEAM 2
Iloilo, Philippines
E

Trim, Stability, Stress


R

C
E

OL
. T H

STUDENT LEARNING MODULE


LEGES
ST

Revision No. 2 Effectivity date: Reviewed by: Approved by:

01 September 2021 QMR President


Multiple Weight Problem:

The calculation for every single weight that is shifted, loaded on, or discharged from the
ship.
In practice, “moments about the keel” are taken to determine the final KG of the ship,
Moments (t-m) = Weight (t) x Distance (m)

If a ship is considered, Moments (t-m) = Displacement (t) x KG (m)

KG(m) = Moments (t-m)


Displacement (t)

If weight is shifted, loaded, or discharged, the moments for each weight are calculated.
These are summed up and divided by the final displacement of the ship to give the final KG.

Example 1:
A ship displaces 10000 t and has a KG of 4.5 m.
The following cargo is worked: Load: 120 t at Kg 6.0 m;
730 t at Kg 3.2 m.
Discharge: 68 t from Kg 2.0 m;
100 t from Kg 6.2 m.
Shift: 86 t from Kg 2.2 m to Kg 6.0 m
Calculate the final KG.

Each weight is multiplied by its KG to give a moment value. The sign of this value (+ or -)
depends on whether the weight is loaded or discharged. In case of the weight that is
shifted, this is treated as two separate weights; one that is discharged; and another of the
same weight load.
Weight (t) KG (m) Moments (t-m)
Ship + 10000 4.50 45000.00
Load + 120 6.00 720.00
Load + 730 3.20 2336.00
Discharge – -68 2.00 -136.00
Discharge – -100 6.20 -620.00
Discharge – -86 2.20 -189.20
Load + 86 6.00 516.00
Final 10682 47626.80

The formula finds the final KG:

KG(m) = Moments(t-m) = 47626.80 = 4.459 m


Displacement(t) 10682

© All Rights Reserved.


Issue No. 1 Page 70 of 262
S E
MT
C
ST. THERESE- MTC COLLEGES SEAM 2
Iloilo, Philippines
E

Trim, Stability, Stress


R

C
E

OL
. T H

STUDENT LEARNING MODULE


LEGES
ST

Revision No. 2 Effectivity date: Reviewed by: Approved by:

01 September 2021 QMR President


Righting Lever or GZ:
Is the horizontal distance measured in meters between the center of gravity (G) and
the vertical line of action of the buoyancy force (Bf) acting through the center of buoyancy
(B1) when the ship is heeled?
Righting lever (GZ) increases to some maximum value and decreases as the ship
progressively heels further.

Moment of Statical Stability or Righting Moment at any angle of the heel can be found:
Righting moment (t-m) = GZ (m) x Displacement (t)

Example:
Calculate the righting moment for a ship with a displacement of 12000 t if the righting lever
(GZ) is 0.46 m when heeled over.

Solution:
Righting moment =GZ x Displacement
RM = 0.46 x 12000
RM = 5520 t-m

Initial Transverse Metacentre (M):


Is the intersection of successive lines of action of buoyancy force (Bf) when the is in
the initial upright condition and subsequently heeled conditions? Is expressed as a height
above the keel in meters or KM. The value of KM is tabulated in the ship‟s hydrostatic data,
and its position varies with draught.

Metacentric Height (GM):


The vertical distance between the ship‟s center of gravity (G) and the initial
transverse metacenter (M). Very important in determining the stability of the ship at a small
angle of the heel.
KM – KG = GM
GM is a positive value. In the normal loaded condition, the initial metacentric height (GM)
should not be less than 0.15 m. Code on Intact Stability for all type of ships covered
by IMO instruments – Chapter 3 section 3.1.2.4

Calculating the Moment of Statical Stability (righting moment) at a small angle of


heel:
In triangle GZM: Sinø = OPP/HYP = GZ/GM
GZ =GM x Sinø
Having found GZ Righting moment = GZ x Displacement

Example:
A ship has a displacement of 8900 t, a corresponding KM of 9.400 m, and a KG of 7.620 m.
a. Calculate the moment of statical stability when the ship is heeled to 5 degrees
b. A 200 t is shifted from the lower hold Kg 4.26 m to the upper deck Kg 12.60 m.
Calculate the moment of statical stability that will now exist if the ship is again
heeled to 5 degrees.

© All Rights Reserved.


Issue No. 1 Page 71 of 262
S E
MT
C
ST. THERESE- MTC COLLEGES SEAM 2
Iloilo, Philippines
E

Trim, Stability, Stress


R

C
E

OL
. T H

STUDENT LEARNING MODULE


LEGES
ST

Revision No. 2 Effectivity date: Reviewed by: Approved by:

01 September 2021 QMR President


Solution:

a. KM 9.400 m
KG 7.620 m
GM 1.780 m

GZ = GM x Sin ø GZ = 1.780 x Sin 5° = 0.15514 m


RM = GZ x Displacement RM = 0.15514 x 8900 = 1380.7 t-m

b. Take moments about the keel:

Weight(t) KG(m) Moments(t-m)


Initial Displ 8900 7.62 67818.0
Discharge -200 4.26 -852.0
Load 200 12.60 2520.0
Final 8900 69486.0

KG = Moments = 69486.0 = 7.807 m Final KG


Displ. 8900.0

GM = KM - KG

KM 9.400
Final KG 7.807
Final GM 1.593 m

GZ = GM X Sin ø GZ = 1.593 x Sin 5° = 0.13884 m


RM = GZ x Displacement RM = 0.13884 x 8900 = 1235.7 t-m

© All Rights Reserved.


Issue No. 1 Page 72 of 262
S E
MT
C
ST. THERESE- MTC COLLEGES SEAM 2
Iloilo, Philippines
E

Trim, Stability, Stress


R

C
E

OL
. T H

STUDENT LEARNING MODULE


LEGES
ST

Revision No. 2 Effectivity date: Reviewed by: Approved by:

01 September 2021 QMR President

References

- Patterson, Chris J. & Jonathan D. Ridley. (2014). Ship stability, powering and
resistance. London: Adlard Coles Nautical.
- Arora, Shiven. (2014). Merchant ship naval architecture. New Delhi, India: Venus
books.
- Barrass, C.B. and Derrett, D.R. (2012). Ship stability for masters and mates. 7th Ed.
USA: Butterworth-Heinemann.
- Eyres, D.J. and Bruce, D.J. (2012). Ship construction. Butterworth. 7th Ed.
Amsterdam: Elsevier.
- Rhodes, Martin A. (2012). Ship stability for masters/mates, Scotland, UK: Witherby
Publishing group
- – IMO. (2017). STCW convention and STCW code including 2010 Manila
Amendments. London: International Maritime
- Organization
- IMO. (2013). Model course 7.03. Officer in – Charge of Navigational Watch. Revised
Ed. London: International Maritime
- Organization.
- de la Calzada, Limic (2012). Ship and ship routines. revised ed. Philippines: MARTA
- Ship Operations and Management. (2012). Scotland, UK: Witherby Publishing Group
Ltd.
- Dokkum, Klaas Van. (2011). Ship knowledge, ship design, construction and
operation, 7th ed. The Netherlands: Dokmar Maritime Publishers.
- Taylor, Dr. D.A. (2011). Merchant ship construction. London: Institute Marine
Engineering Science & Technology.
- House, David J. (2010). Elements of modern ship construction. Glasgow:
Brown, Son & Ferguson Ltd.

© All Rights Reserved.


Issue No. 1 Page 73 of 262
S E
MT
C
ST. THERESE- MTC COLLEGES SEAM 2
Iloilo, Philippines
E

Trim, Stability, Stress


R

C
E

OL
. T H

STUDENT LEARNING MODULE


LEGES
ST

Revision No. 2 Effectivity date: Reviewed by: Approved by:

01 September 2021 QMR President

Checkpoint

Activity 1
True or False

1. GGV is the shift of G: a vertical component while, GGH a horizontal component

2. The value of KM is tabulated in the ship‟s hydrostatic data, and its position varies
with draught.
3. To calculate the final KG, each weight is added by its KG to give a moment value.
4. The sign of this value (+) is used when the weight is loaded.
5. The sign of this value (-) is used when the weight is discharged.

Fill in the blanks.

1. _____ is the horizontal distance measured in meters between the center of gravity
(G) and the buoyancy force's vertical line of action (Bf).
2. _____ is the point of intersection of successive lines of action of buoyancy force (Bf).
3. _____ is the vertical distance between the ship‟s center of gravity (G) and the initial
transverse metacenter (M).
4. The final KG is found by the formula: KG(m) = ______.
5. Righting lever (GZ) increases to some maximum value and then ______ as the ship
progressively heels further.

Do this

Activity 2

1. A ship displaces 15000 t and has an initial KG of 7 m. Calculate the final KG if a


weight of 15 t is moved vertically upwards from 4.5 m to 8.4 KG, respectively.
2. A ship displaces 12000 t and has an initial KG of 6 m. Calculate the final KG if:

a) 1000 t of cargo is loaded into the lower hold at Kg 3 m.

b) 1200 t of cargo is loaded onto the main deck at Kg 10 m.

© All Rights Reserved.


Issue No. 1 Page 74 of 262
S E
MT
C
ST. THERESE- MTC COLLEGES SEAM 2
Iloilo, Philippines
E

Trim, Stability, Stress


R

C
E

OL
. T H

STUDENT LEARNING MODULE


LEGES
ST

Revision No. 2 Effectivity date: Reviewed by: Approved by:

01 September 2021 QMR President

Assessment

1.) A ship displaces 12500 t and has an initial KG of 6 m. Calculate the final KG if:
a) 500 t of cargo is discharged from the lower deck hold at Kg 3 m.
b) 450 t of cargo is discharged from the main deck at Kg 10 m

2.) A 30 MT crate is added 10 M above the keel. Ship displacement is 3500 MT, and KG
is 6 M. What is the new height of the ship‟s center of gravity (KG)?

Problem Solving: (SCORE)


The rubric below will be used in scoring your answer /solution.

Given -- 2
Process/solution -- 5
Answer (correct value & unit -- 3
________
10 Points

© All Rights Reserved.


Issue No. 1 Page 75 of 262
S E
MT
C
ST. THERESE- MTC COLLEGES SEAM 2
Iloilo, Philippines
E

Trim, Stability, Stress


R

C
E

OL
. T H

STUDENT LEARNING MODULE


LEGES
ST

Revision No. 2 Effectivity date: Reviewed by: Approved by:

01 September 2021 QMR President

Lesson 8

Learning Module 8 List and its correction (5 hours)

Competence, Course Outcomes, and Learning Outcomes

Competence:

A-II/1 F3.C2: Maintain seaworthiness of the ship

Course Outcome/s:

Calculate ship stability in compliance with the IMO intact stability


criteria under all conditions of loading

Learning Outcomes:

At the end of the learning module, the student can:

1. Describe the factors which affect the listing of the ship and how it is
corrected

Overview

When there is an uneven distribution of weight in the


transverse direction, the ship heels, sometimes, from collision
or due to bad weather, one side of the ship is damaged, and
water enters compartments in that side, then the ship lists to
that side. On this section will discuss the different conditions of
ship as affected by external
forces or internal forces as it
resulted to the ship
inclination either port or
starboard in any case.
Calculation of ship inclination
as a result of an even distribution of cargo during loading.
At the last part of the lesson students will perform the
provided check point.

© All Rights Reserved.


Issue No. 1 Page 76 of 262
S E
MT
C
ST. THERESE- MTC COLLEGES SEAM 2
Iloilo, Philippines
E

Trim, Stability, Stress


R

C
E

OL
. T H

STUDENT LEARNING MODULE


LEGES
ST

Revision No. 2 Effectivity date: Reviewed by: Approved by:

01 September 2021 QMR President

Discussion

2. Key Concept

The list's angle is the degree to which a vessel's heels (leans or tilts) to either port or
starboard (often caused by uneven loading or flooding) causes it to heel to one side.

© All Rights Reserved.


Issue No. 1 Page 77 of 262
S E
MT
C
ST. THERESE- MTC COLLEGES SEAM 2
Iloilo, Philippines
E

Trim, Stability, Stress


R

C
E

OL
. T H

STUDENT LEARNING MODULE


LEGES
ST

Revision No. 2 Effectivity date: Reviewed by: Approved by:

01 September 2021 QMR President


Now let a weight already on board
the ship be shifted transversely
such that G moves to G1. (figure a)
This will produce a listing moment
of W x GG1, and the ship will list
until G1, and the center of
buoyancy is in the same vertical
line (figure B)

In this position, G1 will also lie


vertically under M so long as the
list's angle is small. Therefore, if
the final positions of the meta-
center and the center of gravity are known, the final list can be found, using trigonometry,
in the triangle GG1M which is right-angled at G

The list is removed by adding weight to the opposite side of removing weights from side of
the list. (This is so if you are sure that the list is due to the G off the centerline)

© All Rights Reserved.


Issue No. 1 Page 78 of 262
S E
MT
C
ST. THERESE- MTC COLLEGES SEAM 2
Iloilo, Philippines
E

Trim, Stability, Stress


R

C
E

OL
. T H

STUDENT LEARNING MODULE


LEGES
ST

Revision No. 2 Effectivity date: Reviewed by: Approved by:

01 September 2021 QMR President

Example:
When the weight is shifted transversely, the ship‟s center of gravity will also shift
transversely from G to G1. The ship will then list θ degrees to bring G1 vertically under M the
metacenter.

GG1 = w x d = 60 x 12 GG1 or GGH Horizontal component (List)


W 6000

GG1 = 0.12 m

GM = KM – KG = 7.3 – 6.7 = 0.6m

LIST AND ITS CORRECTION

In triangle GG1M:

tan θ = OPP = GG1


ADJ GM

= 0.12 = 0.20
0.60

= 11.30990

Ans. List θ is = 11°18.5’

© All Rights Reserved.


Issue No. 1 Page 79 of 262
S E
MT
C
ST. THERESE- MTC COLLEGES SEAM 2
Iloilo, Philippines
E

Trim, Stability, Stress


R

C
E

OL
. T H

STUDENT LEARNING MODULE


LEGES
ST

Revision No. 2 Effectivity date: Reviewed by: Approved by:

01 September 2021 QMR President

Loading and Discharging Multiple Problem:

© All Rights Reserved.


Issue No. 1 Page 80 of 262
S E
MT
C
ST. THERESE- MTC COLLEGES SEAM 2
Iloilo, Philippines
E

Trim, Stability, Stress


R

C
E

OL
. T H

STUDENT LEARNING MODULE


LEGES
ST

Revision No. 2 Effectivity date: Reviewed by: Approved by:

01 September 2021 QMR President

Calculating the list due to a single weight being loaded or discharge

Weight is loaded and discharge, then both the vertical and horizontal components of the G
must be considered, and the final GM must be calculating the final list.

Rules:
Weight is loaded G will move directly towards the center of gravity of the loaded weight.
Weight is discharged G will move directly away from the center of gravity of the discharged
weight.

© All Rights Reserved.


Issue No. 1 Page 81 of 262
S E
MT
C
ST. THERESE- MTC COLLEGES SEAM 2
Iloilo, Philippines
E

Trim, Stability, Stress


R

C
E

OL
. T H

STUDENT LEARNING MODULE


LEGES
ST

Revision No. 2 Effectivity date: Reviewed by: Approved by:

01 September 2021 QMR President

Procedure for single weight load/discharge problem:

1. Calculate GGV using: GGV = w x d


W±w
„d‟ is the vertical distance between G of the ship and g of the loaded/discharged
weight.
2. Apply GGV to the ship‟s initial KG to find the final KG
3. Calculate the final GM using: GM = KM – KG
4. Calculate GGV using: GGH = w x d
W±w
„d‟ is the horizontal distance between G of the ship and g of the loaded/discharged
weight.
5. Using the formula: Tan ø LIST = GGH Calculate List
GM

Example 3:
A ship initially upright displaces 16000 t and has KG 4.6 m and KM 6.5 m. a weight of 80 t is
loaded on deck at Kg 10.2, 6.2 m off the centerline to starboard. Calculate the final list.
Assume KM remains constant.

Solution
GGv = w x d GGv = 80 x (10.2 – 4.6) =0.069 m
W+w 6400 +80

Initial KG 4.600 m KM 6.500 m


GGv (up) 0.069 m Final KG 4.669 m
Final KG 4.669 m Final GM 1.831 m

GGH = w x d GGH = 80 x 6.2 =0.077 m


W+w 6400 +80

Tan ø LIST = GGH = 0.077 = 0.04205


GMFINAL 1.831 List = 2.4° Stbd.

Example 4
A ship initially upright displaces 14480 t and has a KG 8.82 m and KM 10.96 m. a weight of
240 t is discharged from a lower hold Kg 3.6 m, 2.8 m off the centerline to port. Calculate
the final list. Assume KM remains constant.

Solution

GGv = w x d GGv = 210 x (8.82 – 3.6) =0.088 m


W-w 14480 +240

© All Rights Reserved.


Issue No. 1 Page 82 of 262
S E
MT
C
ST. THERESE- MTC COLLEGES SEAM 2
Iloilo, Philippines
E

Trim, Stability, Stress


R

C
E

OL
. T H

STUDENT LEARNING MODULE


LEGES
ST

Revision No. 2 Effectivity date: Reviewed by: Approved by:

01 September 2021 QMR President

Initial KG 8.820 m KM 10.960 m


GGv (up) 0.088 m Final KG 8.908 m
Final KG 8.908 m Final GM 2.052 m

GGH = w x d GGH = 240 x 2.8 =0.047 m


W-w 14480 - 240

Tan ø LIST = GGH = 0.047 = 0.02290


GMFINAL 2.052 List = 1.3° Stbd.

Shifting a Weight Already on Board to Bring a Listed Ship Upright

For a ship to be upright: Port Moments = Starboard Moments where


G must be on the centerline.
A ship that is listed will have G off the centerline by a distance GGH
as shown.
Consider the formula for a shift of weight: GGH = w x d
W
Rearranging this gives: GGH x W = w x d
(GGH x W) represents the listing moments that the ship initially has.
(w x d) represents the moments required to equal (GGH x W) if the
ship is to complete upright.

Example 5
A ship has a displacement of 12000 t and is initially listed 2° to
starboard. If the ship's KG is 11.60 and the KM is 12.00 m, how
much ballast to a port-side ballast tank through a distance of 16.00
m?

The ship is initially listed as starboard.

KM 12.00 m
KG 11.60 m
GM 0.40 m

Tan ø LIST = GGH Tan 2° = GGH


GM 0.40

© All Rights Reserved.


Issue No. 1 Page 83 of 262
S E
MT
C
ST. THERESE- MTC COLLEGES SEAM 2
Iloilo, Philippines
E

Trim, Stability, Stress


R

C
E

OL
. T H

STUDENT LEARNING MODULE


LEGES
ST

Revision No. 2 Effectivity date: Reviewed by: Approved by:

01 September 2021 QMR President

GGH = Tan 2° x 0.40 = 0.014 m


G is off the centerline to starboard by 0.014 m.

Required port moments to counter list (w x d) must equal initial starboard listing moments
(GGH x W).
GGH x W = w x d
0.014 x 12000 = w x 16.00
168 = 16w
W = 168 / 16
w = 10.5 tonnes to transfer

Multiple Weight Problems – Ship Initially Upright


In practice, list problems are solved by taking moments about the keel to determine the final
KG and the final GM; and then taking moments about the centerline to determine GGH.

The procedure is as follows:

1. Take moments about the keel to determine the final KG:


Final KG = Sum of moments about keel (t -m)
Final Displacement(t)
2. Calculate the final GM: GM = KM – KG
3. Take moments about the centerline to calculate the final distance that G is off the
centerline, GGH:
GGH = Sum of moments about the centerline (t -m)
Final Displacement(t)
Calculate the list: Tan ø LIST = GGH
GMFINAL

Example 6
A ship displaces 8000 tonnes, KG 7.60 m, and is initially upright.
The following cargo is worked:

Load: 300 t at Kg 0.60 m, 6.1 m to port of CL;


250 t at Kg 6.10 m, 7.6 m to stbd of CL;
Disch: 50 t from Kg 1.20 m, 4.6 m to pot of CL;
500 t from Kg 12.60 m, 4.6 m to stbd of CL.

Calculate the list's final angle to complete cargo if the KM for the final displacement is o.36
m.

Solution
Take moments about the keel to determine the final KG and GM.

Weight (t) KG (m) Moments (t-m)


Initial display. 8000 7.60 60800.00

© All Rights Reserved.


Issue No. 1 Page 84 of 262
S E
MT
C
ST. THERESE- MTC COLLEGES SEAM 2
Iloilo, Philippines
E

Trim, Stability, Stress


R

C
E

OL
. T H

STUDENT LEARNING MODULE


LEGES
ST

Revision No. 2 Effectivity date: Reviewed by: Approved by:

01 September 2021 QMR President


Load 300 0.60 180.00
Load 250 6.10 1525.00
Discharge -50 1.20 -60.00
Discharge -500 12.60 -6300.00
Final 8000 7.018 56145.00

KM 9.360
KG 7.018
GM 2.342

Take moments about the centerline to determine GGH


Weight (t) Dist. off CL Port Moments Stbd Moments
(t-m) (t-m)
8000 0.00 0.0 0.0
300 6.10 1830.0
250 7.60 1900.0
50 4.60 230.0
500 4.60 2300.0
4130.0 2130.0
2130.0
2000.0
GGH= = =0.250 m
Calculated the final list.
Tan LIST= = =0.10675
Final list= 6.1° Port

Multiple Weight Problems- Ship Initially Listed

If a ship is initially listed, G must be of the centerline as shown.

Since: Tan LIST=

then: GGH= Tan LISTx GM

Where GGH is the initial listing lever to be incorporated into the


moment's table for the ship, consider the following example.

Example 7
A ship has a displacement of 15000t, KG 8.6 m, Km 9.4, and is listed
6° to starboard. Cargo is worked as follows:
Load 150 t at Kg 7.6 m, 5.0 m to port of CL:
Load 305 t at Kg 8.0 m, on the CL:
Load 95 t at Kg 8.0 m, 4.2 m to starboard of CL.

© All Rights Reserved.


Issue No. 1 Page 85 of 262
S E
MT
C
ST. THERESE- MTC COLLEGES SEAM 2
Iloilo, Philippines
E

Trim, Stability, Stress


R

C
E

OL
. T H

STUDENT LEARNING MODULE


LEGES
ST

Revision No. 2 Effectivity date: Reviewed by: Approved by:

01 September 2021 QMR President


Calculate the final angle of the list.
(Assume KM remains constant)

Solution
Calculate initial GM.

KM 9.400 m Calculate GGH using: GGH=Tan x GM


Initial KG 8.600m GGH=Tan 6°x 0.800 =0.084m.
Initial GM 0.800m

Take moments about the keel to determine the final KG and GM.
Weight (t) KG (m) Moments (t-m)
Initial display. 15000 8.60 129000.0
Load 150 7.60 1140.0
Load 305 8.00 2440.0
Load 95 8.00 760.0
Final 15550 8.575 133340.0
KM 9.400
KG 8.575
GM 0.825

Take moments about the centerline to calculate the final GGH.


Weight (t) Dist off CL (m) Port Moments Starboard Moments
(t-m) (t-m)
15000 0.084 1260.0
150 5.000 750.0
305 0.000 0.0 0.0
95 4.200 399.0
750.0 1659.0
750.0
909.0

GGH= = =0.058 m
Calculate the final list.
Tan = = =0.07030

Final list= 4.0° Stbd.

© All Rights Reserved.


Issue No. 1 Page 86 of 262
S E
MT
C
ST. THERESE- MTC COLLEGES SEAM 2
Iloilo, Philippines
E

Trim, Stability, Stress


R

C
E

OL
. T H

STUDENT LEARNING MODULE


LEGES
ST

Revision No. 2 Effectivity date: Reviewed by: Approved by:

01 September 2021 QMR President


Loading Weight about the Centre Line to Complete upright

A common question arises where the ship is near completion of loading. The remaining
cargo has to be distributed between two compartments that either side of the centerline so
that the ship completes upright.

To complete upright: Port moments must equally starboard moments!

There are two methods of approach to this type of problem, as shown in the next example.

Example 8 (Method 1)

From the following details calculate the final GM and the amount of cargo to load in each space
so that the ship will complete upright:

Initial displacement 18000 t, KG 8.80 m, KM 9.40 m and listed 3° starboard. 400 tonnes of cargo
remains to be loaded where space is available in tween deck Kg 10.5 m, 7.0 m to port of CL and
10.0 m to starboard of CL. (Assume KM remains constant).

Solution
Calculate initial GM.
KM 9.400
Initial KG 8.800
Initial GM 0.600

Calculate GGH using GGH = Tan øLIST x GM

GGH = Tan 3° x 0.600 = 0.031m.

Take moments about the keel to determine the final KG and GM (note that all 400 t of cargo
is loaded at Kg 10.5 m, so treat as a single weight).

Weight (t) KG (m) Moments (t-m)


Initial 18000 8.80 158400.0
display. KM 9.400
Load 400 10.50 4200.0 KG 8.837
Final 18400 8.835 162600.0 GM 0.563

Taking moments about the centerline load all 400 t in the port side.

Weight (t) Disc off CL Port Moments Starboard


(t-m) Moments (t-m)
18000 0.031 558.0
400 7.000 2800.0
2800.0 558.0
© All Rights Reserved.
Issue No. 1 Page 87 of 262
S E
MT
C
ST. THERESE- MTC COLLEGES SEAM 2
Iloilo, Philippines
E

Trim, Stability, Stress


R

C
E

OL
. T H

STUDENT LEARNING MODULE


LEGES
ST

Revision No. 2 Effectivity date: Reviewed by: Approved by:

01 September 2021 QMR President


558.0
2242.0

If all 400 t were loaded into the port side space, the ship would complete with an excess of
2242 t-m moments to port. Therefore, some 400 t must now be shifted to space on the
starboard side (a distance of 17.0 m).

2242 = w x d
2242 = w x (7.0 + 10.0)
2242 = 17w
w = 2242 = 131.9 t to shift from port to starboard
17
To complete upright:
Load 400.0 – 131.9
=268.1 t port Load 131.9 t Starboard

Solution (Method 2)
Calculate initial GM.

KM 9.400
Initial KG 8.800
Initial GM 0.600

Calculate GGH using: GGH = Tan øLIST x GM


GGH = Tan 3° x 0.600 = 0.031 m.

Take moments about the keel to determine the final KG and GM (note that all 400 t of cargo
is loaded at Kg 10.5 m, so treat as a single weight!).

Weight (t) KG (m) Moments (t-m)


Initial 18000 8.80 158400.0
display. KM 9.400
Load 400 10.50 4200.0 KG 8.837
Final 18400 8.837 162.600.0 GM 0.563

Taking moments about the centre line: Let x = cargo to load port; (400 – x) = cargo to load
to starboard.

Weight (t) Disc off CL Port Moments Starboard


(t-m) Moments (t-m)
18000 0.031 558.0
X 7.000 7x
(400-x) 10.000 (4000 – 10x)
7x 558 + (4000-10x)

© All Rights Reserved.


Issue No. 1 Page 88 of 262
S E
MT
C
ST. THERESE- MTC COLLEGES SEAM 2
Iloilo, Philippines
E

Trim, Stability, Stress


R

C
E

OL
. T H

STUDENT LEARNING MODULE


LEGES
ST

Revision No. 2 Effectivity date: Reviewed by: Approved by:

01 September 2021 QMR President


To complete upright:

Port moments must equally starboard moments.


Therefore: 7x = 558 + (4000 – 10x)
7x = 558 + 4000 – 10x
7x + 10x = 558 + 4000
17x = 4558
X = 4558
17
X = 268.1 t to port
400 – 268.1 = 131.9 t to starboard
List and Free Surface Effect

Consider figure 11.8

The basic list triangle is GGHM. GM is the solid metacentric


height; the GM would exist if the ship had no slack tanks.

GGH is the distance that G is off the centerline.

GGV is the virtual rise of G due to tank-free surfaces. Since GM


is reduced to GVM (the Fluid GM), the list's angle has
increased for the same distance that G is of the centerline
(GGH).

The greater the free surface moments/free surface effect, the greater will be list
for the same listing moments.

Example 9
A ship displaces 13200 t, KG 10.2 m, and is initially upright. Ballast water RD 1.025 is run
into a rectangular DB tank length 24 m, breadth 10 m to a sounding of 4.00 m. If the Kg of
the ballast water is 2.00 m and its transverse center of gravity (TCG) is 5.00 m to the
starboard of the centerline, calculate the final angle of the list:
a. Assuming no free surface moments:
b. Accounting for free surface moments.
Assume KM for the final displacement is 11.64 m.

Solution
mass of ballast water loaded = 24 x 10 x 4 x 1.025 = 984 t
a. (assuming no FSMs)
Weight (t) KG (m) Moments (t-m) KM 11.640
Initial displ. 13200 10.20 134640.0 KG 9.631
Load 984 2.00 1968.0 GM 2.009
Final 14184 9.631 136608.0

Taking moments about the keel to determine final KG and GM.


KG = Moments / Displacement = 136608 / 14184 = 9.631

© All Rights Reserved.


Issue No. 1 Page 89 of 262
S E
MT
C
ST. THERESE- MTC COLLEGES SEAM 2
Iloilo, Philippines
E

Trim, Stability, Stress


R

C
E

OL
. T H

STUDENT LEARNING MODULE


LEGES
ST

Revision No. 2 Effectivity date: Reviewed by: Approved by:

01 September 2021 QMR President

Taking moments about the center of the line, calculate GGH.

Weight (t) Disc off CL Port Moments Starboard


(t-m) Moments (t-m)
13200 0.000 0.0 0.0
984 5.000 4920.0
0.0 4920.0

GGH = Net listing moments = 4920 = 0.347


Final displacement 14184

Calculate the final list.

Tan øLIST = GGH = 0.347 = 0.17272


GMFINAL 2.009

Final list = 9.8° Stbd

b. (including FSMs)

FSMs = lb3 x dt = 24 x 103 x 1.025 = 2050 t-m


12 12

Taking moments about the keel to determine final KG and GM.

Weight (t) KG (m) Moments (t-m)


Initial displ. 13200 10.20 134640.0
Load 984 2.00 1968.0 KM 11.640
FSMs 2050 KG 9.776
Final 14184 9.776 138658.0 GM 1.864

The value of GGH is not affected by free surface moments.

Calculate the final list.

Tan øLIST = GGH = 0.347 = 0. 18616 List = 10.5" Stbd


GMFINAL 1.864

When calculating the list, free surface effects should always be accounted for; they will
always cause an increase the ship list!

© All Rights Reserved.


Issue No. 1 Page 90 of 262
S E
MT
C
ST. THERESE- MTC COLLEGES SEAM 2
Iloilo, Philippines
E

Trim, Stability, Stress


R

C
E

OL
. T H

STUDENT LEARNING MODULE


LEGES
ST

Revision No. 2 Effectivity date: Reviewed by: Approved by:

01 September 2021 QMR President


Example 10

A ship displacing 7790 t, KG 7.57 m, KM 8.12 m is floating upright. A double bottom tank of
the rectangular cross-section is divided into two equal parts, each 16 m long, 6.9 m wide,
and 1.6 m deep.

In the upright condition, the port side is full of diesel oil (RD 0.88), and the starboard side is
empty.

Calculate the list's resulting angle when half of this oil is transferred to the starboard side of
the dark.

Solution
Assume that the bottom of the double bottom tank is at the keel.
Mass of oil transferred = 16 x 6.9 x 0.8 = 77.7 tonnes
Oil is moved down from Kg 1.2 m to Kg 0.4 m.

FSMs = lb3 x dt x 2 tanks


12

FSMs = 16 x 6.93 x 0.88 = 770.9 t-m


12

Taking moments about the keel, calculate the final KG and hence the final GM.

Weight (t) KG (m) Moments (t-m)


Initial displ. 7790.0 7.57 58970.30
Disch. -77.7 1.20 -93.24
Load 77.7 0.40 31.08 KM 8.120
FSMs 770.90 KG 7.661
Final 7790.0 7.661 59679.04 GM 0.459

© All Rights Reserved.


Issue No. 1 Page 91 of 262
S E
MT
C
ST. THERESE- MTC COLLEGES SEAM 2
Iloilo, Philippines
E

Trim, Stability, Stress


R

C
E

OL
. T H

STUDENT LEARNING MODULE


LEGES
ST

Revision No. 2 Effectivity date: Reviewed by: Approved by:

01 September 2021 QMR President


Oil is transferred to starboard through a distance of 6.9 m.

GGH = w x d = 77.7 x 6.9 = 0.069 m


W 7790

Tan øLIST = GGH = 0.069 = 0.15033 List = 8.5" Stbd.


GMFINAL 0.459

References

– 3G E-Learning. (2017). Essential seamanship. USA: 3G E-Learning LLC.


– 3G. E-Learning (2017). Ship and operation’s management. USA: 3G E
Learning LLC
- Patterson, Chris J. & Jonathan D. Ridley. (2014). Ship stability, powering and
resistance. London: Adlard Coles Nautical.
- Arora, Shiven. (2014). Merchant ship naval architecture. New Delhi, India:
Venus books.
- Barrass, C.B. and Derrett, D.R. (2012). Ship stability for masters and mates. 7th
Ed. USA: Butterworth-Heinemann.

Checkpoint
Activity 1
Fill in the blanks.
1. The _______ is the degree to which a vessel heels (leans or tilts) to either port or
starboard.
2. ______ represents the listing moments that the ship initially has.
3. ______ represents the moments required to equal to GGH x W if the ship is to
complete upright.
4. ______ is the distance that G is off the centerline.
5. ______ is the virtual rise of G due to tank-free surfaces.

© All Rights Reserved.


Issue No. 1 Page 92 of 262
S E
MT
C
ST. THERESE- MTC COLLEGES SEAM 2
Iloilo, Philippines
E

Trim, Stability, Stress


R

C
E

OL
. T H

STUDENT LEARNING MODULE


LEGES
ST

Revision No. 2 Effectivity date: Reviewed by: Approved by:

01 September 2021 QMR President

True or False
1. Weight is loaded and discharge, then both the vertical and horizontal components of
the shift of G must be considered and the final GM.
2. Weight is loaded G will move directly away from the center of gravity of the loaded
weight.
3. Weight is discharged G will move directly away from the center of gravity of the
discharged weight.
4. For a ship to be upright, the Port Moments must not be equal to Starboard Moments,
where G must be on the centerline.
5. The greater the free surface moments or free surface effect, the greater will be list
for the same listing moments.

© All Rights Reserved.


Issue No. 1 Page 93 of 262
S E
MT
C
ST. THERESE- MTC COLLEGES SEAM 2
Iloilo, Philippines
E

Trim, Stability, Stress


R

C
E

OL
. T H

STUDENT LEARNING MODULE


LEGES
ST

Revision No. 2 Effectivity date: Reviewed by: Approved by:

01 September 2021 QMR President

Do this

Activity 2

1. A ship of 5000 tonnes displacement has KG 4.2 m and KM 4.5 m and is listed 5 degrees
to port. Assuming that the KM remains constant, find the final list if 80 tonnes of
bunkers are loaded in the No. 2 starboard tank whose center of gravity is 1 meter above
the keel and 4 meters out from the centerline.
2. A ship of 8000 tonnes displacement has a GM 0.5 m. A quantity of grain in the hold,
estimated at 80 tonnes, shifts, and, as a result, the center of gravity of this grain moves
6. m horizontally and 1.5 m vertically. Find the resultant list.

Assessment

A ship displaces 13500 t, KG 10.16 m, and is initially upright. Ballast water RD 1.025 is run
into a rectangular DB tank length 20 m, breadth 12 m to a sounding of 4.00 m. If the Kg of
the ballast water is 2.00 m and its transverse center of gravity (TCG) is 4.00 m to the
starboard of the centerline, calculate the final angle of the list:
a. Assuming no free surface moments:
b. Accounting for free surface moments.
Assume KM for the final displacement is 11.60 m.

© All Rights Reserved.


Issue No. 1 Page 94 of 262
S E
MT
C
ST. THERESE- MTC COLLEGES SEAM 2
Iloilo, Philippines
E

Trim, Stability, Stress


R

C
E

OL
. T H

STUDENT LEARNING MODULE


LEGES
ST

Revision No. 2 Effectivity date: Reviewed by: Approved by:

01 September 2021 QMR President

Lesson 9

Learning Module 9.1 Effect of slack tanks (5 hours)

Competence, Course Outcomes, and Learning Outcomes

Competence:

A-II/1 F3.C2: Maintain seaworthiness of the ship

Course Outcome/s:

Calculate ship stability in compliance with the IMO intact stability


criteria under all conditions of loading

Learning Outcomes:

At the end of the end of learning module, the student can:

1. Show by a computation that if a tank is full of liquid, its effect on the ship‟s
Center of gravity's position is the same as if the liquid were a solid of the
same mass.

Overview

When any tank or a compartment is partially filled,


the liquid's motion (due to the ship‟s rolling and
pitching motions) will reduce its stability, the ship is
inclined, the tank's liquid shifts to the tank's lower
side.
The ship tank extends from the bottom to the tank
top and is only partially filled, the ship is in on
upright position, the liquid's free surface in the tank
is shown as in fig. AA1. When the ship inclines to a
certain angle of the heel (say „theta‟ Ɵ), the liquid's
free surface now changes to TT1. Also, the center of

© All Rights Reserved.


Issue No. 1 Page 95 of 262
S E
MT
C
ST. THERESE- MTC COLLEGES SEAM 2
Iloilo, Philippines
E

Trim, Stability, Stress


R

C
E

OL
. T H

STUDENT LEARNING MODULE


LEGES
ST

Revision No. 2 Effectivity date: Reviewed by: Approved by:

01 September 2021 QMR President


buoyancy shifts from „B‟ to „B1‟.

What happens as a result of this? Do note that when the liquid in the tank is transferred to
the lower side, the liquid volume within the wedge between points „A‟ and „T‟ has now
shifted to the lower side between points A1 and T1. The center of gravity of that volume of
liquid has shifted from „g‟ to „g1‟. On this section will have the understanding of shift of liquid
cargo, computation of the shift of center of gravity. At the last part of the lesson student will
perform the checkpoint.

Discussion

3. Key Concept EFFECT OF SLACK TANKS

© All Rights Reserved.


Issue No. 1 Page 96 of 262
S E
MT
C
ST. THERESE- MTC COLLEGES SEAM 2
Iloilo, Philippines
E

Trim, Stability, Stress


R

C
E

OL
. T H

STUDENT LEARNING MODULE


LEGES
ST

Revision No. 2 Effectivity date: Reviewed by: Approved by:

01 September 2021 QMR President


When the tank is filled with a liquid, the liquid cannot move within the tank when the
ship heels. For this reason, as far as stability is concerned, the liquid may be considered as a
static weight having its center of gravity at the center of gravity of the liquid within the tank.
Now consider the same ship floating at the
same draft and having the same KG, but increase the
tank's depth so that liquid now only partially fills, as
shown in the next figure.
When the ship heels, as shown, the liquid flows
to the tank's low side such that it's center of gravity
shifts from g to g1. This will cause the ship‟s center of
gravity to shift from G to G1, parallel to gg1.
This indicates that the free surface's effect
reduces the sufficient metacentric height from GM to
GvM.
GGv is, therefore, the virtual loss of GM due to the
free surface.
Any loss in GM is a loss instability.
If a free surface is created in a ship with a small initial metacentric height, GM's virtual loss
due to the free surface may result in a negative metacentric height.
This would cause the ship to take up an angle of loll, which may be dangerous and,
in any case, is undesirable.
This should be borne in mind when considering running water ballast into tanks to
correct an angle of loll or increase the GM.
Until the tank is full, there will be a virtual loss of GM due to the liquid's free surface
effect.
The increase in KG is affected mainly by the free surface's breadth and is not
dependent upon the tank's liquid mass.
In tankers, the tanks are often constructed with a longitudinal subdivision to reduce
the free surface's breadth.

Free Surface Effect and the Loss of Transverse Statical Stability


Consider the ship shown with a partially filled tank; imagine the tank's liquid is
frozen, and the ship is heeled to a small angle.
In the heeled condition, GZ is the righting lever. Because the liquid is frozen, it acts as a
static weight and does not move.
Consider what will happen if the liquid in the tank thaws out and is then free to move
as the ship heels, as would typically be the case. In the initial upright condition, everything
appears normal. But note what happens as an external force again heels the ship to the
same small angle of inclination.
A wedge of the liquid is transferred to the low side of the ship (gg1). Since weight has
shifted, G moves parallel and in the same direction as the weight shift (GG1). This causes
the righting lever to be reduced from GZ to G1Z1. The righting lever G1Z1 is the same as the
GZ that would have existed had G been raised to GV.
GGV represents the virtual rise of G that results from the free surface effect of the
slack tank. (G does not rise, but the movement of the liquid in the tank has the same effect
on GZ values as if G had been caused to rise-hence the term „virtual rise of G‟)

© All Rights Reserved.


Issue No. 1 Page 97 of 262
S E
MT
C
ST. THERESE- MTC COLLEGES SEAM 2
Iloilo, Philippines
E

Trim, Stability, Stress


R

C
E

OL
. T H

STUDENT LEARNING MODULE


LEGES
ST

Revision No. 2 Effectivity date: Reviewed by: Approved by:

01 September 2021 QMR President

GM is termed the solid GM


GVM is termed the effective or fluid GM
When calculating the GM of a ship, the effects of free surfaces in slack tanks must be
considered. The loss of GZ will be more significant as the number of slack tanks increases;
all slack tanks' cumulative effect must be accounted for. It is always the fluid GM that must
be determined to reduce GZ values that arise from fluid movement within the ship as it is
heeled. When returning to the upright condition, the center of gravity (G1) will move back to
G's original position as the tank's liquid finds its lever.

Calculating the effect of the free surface in a rectangular-shaped tank.


For a tank that has a rectangular free surface, the virtual rise of G in meters can be
calculated by,

GGV = lb3 / 12V x dt / ds


GGV is the virtual rise of G in meters
l is the tank length
b is the tank breadth
dt is the density of the liquid in the tank
ds is the density of the water in which the ship floats
V is the volume of displacement of the ship

Displacement = Volume of Displacement x Density


W = V x ds
GGV = lb3 x dt
12W

Example:
A ship has an initial displacement of 10500 t and KG 7.60 m. A rectangular cargo oil tank of
length 30 m breadth 20 m is partially filled with 9600 t of oil (RD 0.86). If the KG of oil is 8.0
m, calculate the effective GM if the KM for the final displacement is 8.8 m.

Solution:
1. Taking moments about the keel, calculate the new solid KG.
Weight (t) KG (m) Moments (t-m)
Initial displ. 10500 7.600 79800
Cargo oil 9600 8.000 76800
Final 20100 7.791 156600

New Solid KG = total moments / Total Weight = 156600 / 20100 = 7. 791 m.

© All Rights Reserved.


Issue No. 1 Page 98 of 262
S E
MT
C
ST. THERESE- MTC COLLEGES SEAM 2
Iloilo, Philippines
E

Trim, Stability, Stress


R

C
E

OL
. T H

STUDENT LEARNING MODULE


LEGES
ST

Revision No. 2 Effectivity date: Reviewed by: Approved by:

01 September 2021 QMR President

2. Calculate the effect of free surface

3. Calculate the solid GM and then apply the free surface correction to obtain the fluid
GM.
KM 8.800
Solid KG 7.791
Solid GM 1.009
FSE (GGV) 0.856
Fluid GM 0.153

Free surface moments for an assumed density value, Method 1

Consider the extract from a tank-sounding table shown, paying particular attention.
Table for cargo oil Tank:
Tank 2CO.Stbd Cargo oil Density 0.740

Sounding Weight LCG TCG VCG FSMs


(cms) (t) (m foap) (m) (m) (t-m)
0 0.00 78.145 0.063s 1.503 0.0
25 13.00 78.290 2.963s 1.656 269.6
50 30.83 78.345 3.242s 1.785 300.3
75 49.31 78.366 3.365s 1.914 333.3
100 68.44 78.379 3.457s 2.044 368.2
125 87.92 78.389 3.525s 2.174 370.9
150 107.46 78.398 3.569s 2.302 373.7
175 127.05 78.406 3.602s 2.430 376.5
200 146.70 78.414 3.628s 2.557 379.3
225 166.42 78.421 3.648s 2.684 382.1

Free Surface Moments (FSMs) in t-m are tabulated for an assumed liquid density of 0.740
VCG (vertical center of gravity or Kg) indicates the vertical position of oil within the ship
meters above the keel for the appropriate sounding.

Example:
A ship displaces 5400 t and has a KG of 7.860 m. No. 2 cargo oil tank Stbd. Is filled to a
sounding of 150 cms with cargo oil RD 0.740. Calculate the final sufficient KG and GM if the
KM for the final condition is 8.000m.
(Use the sounding table extract given)

Solution:

© All Rights Reserved.


Issue No. 1 Page 99 of 262
S E
MT
C
ST. THERESE- MTC COLLEGES SEAM 2
Iloilo, Philippines
E

Trim, Stability, Stress


R

C
E

OL
. T H

STUDENT LEARNING MODULE


LEGES
ST

Revision No. 2 Effectivity date: Reviewed by: Approved by:

01 September 2021 QMR President


Obtain cargo data from a table.
Tank 2CO.Stbd Cargo oil Density 0.740

Sounding Weight LCG TCG VCG FSMs


(cms) (t) (m foap) (m) (m) (t-m)
0 0.00 78.145 0.063s 1.503 0.0
25 13.00 78.290 2.963s 1.656 269.6
50 30.83 78.345 3.242s 1.785 300.3
75 49.31 78.366 3.365s 1.914 333.3
100 68.44 78.379 3.457s 2.044 368.2
125 87.92 78.389 3.525s 2.174 370.9
150 107.46 78.398 3.569s 2.302 373.7
175 127.05 78.406 3.602s 2.430 376.5
200 146.70 78.414 3.628s 2.557 379.3
225 166.42 78.421 3.648s 2.684 382.1

Calculate the final effective KG and hence the final effective GM


Weight (t) KG (m) Moments (t-m)
Ship 5400.00 7.860 42444.0
Cargo oil 107.46 2.302 247.4
FSMs 373.7
Final 5507.45 7.819 43065.1

Final Effective KG = Final Moments / Final Weight


= 43065.1 / 5507.46 = 7.819 m

KM 8.000
Fluid KG 7.819
Fluid GM 0.181

For intermediate sounding, interpolation is necessary.


Both mass(weight) and the value of the FSMs are directly proportional to density.

Example:
A ship displaces 5400 t and has a KG of 7.860 m. No. 2 cargo oil tank Stbd. Is filled to a
sounding of 150 cms with salt ballast water RD 1.025. Calculate the final effective KG and
GM if the KM for the final condition is 8.000m.
(Use the sounding table extract given)

© All Rights Reserved.


Issue No. 1 Page 100 of 262
S E
MT
C
ST. THERESE- MTC COLLEGES SEAM 2
Iloilo, Philippines
E

Trim, Stability, Stress


R

C
E

OL
. T H

STUDENT LEARNING MODULE


LEGES
ST

Revision No. 2 Effectivity date: Reviewed by: Approved by:

01 September 2021 QMR President


Solution:
Obtain cargo data from a table.
Tank 2CO.Stbd Cargo oil Density 0.740

Sounding Weight LCG TCG VCG FSMs


(cms) (t) (m foap) (m) (m) (t-m)
0 0.00 78.145 0.063s 1.503 0.0
25 13.00 78.290 2.963s 1.656 269.6
50 30.83 78.345 3.242s 1.785 300.3
75 49.31 78.366 3.365s 1.914 333.3
100 68.44 78.379 3.457s 2.044 368.2
125 87.92 78.389 3.525s 2.174 370.9
150 107.46 78.398 3.569s 2.302 373.7
175 127.05 78.406 3.602s 2.430 376.5
200 146.70 78.414 3.628s 2.557 379.3
225 166.42 78.421 3.648s 2.684 382.1

The actual mass of saltwater ballast loaded is found by:

107.46 x 1.025 / 0.740 = 148.85 t

Actual FSMs for the salt water ballast loaded is found by:

373.7 x 1.025 / 0.740 = 517.625 t-m

The Kg of the liquid is as before – not being affected by density.

Calculate the final effective KG and hence the final effective GM precisely as before.
Weight (t) KG (m) Moments (t-m)
Ship 5400.00 7.860 42444.0
SW Ballast 148.85 2.302 342.6
FSMs 517.6
Final 5548.85 7.804 43304.3

Final Effective KG = Final Moments / Final Weight


= 43304.30 / 5548.85 = 7.804 m
Final Effective KG or Fluid KG

KM 8.000
Fluid KG 7.804
Fluid GM 0.196

© All Rights Reserved.


Issue No. 1 Page 101 of 262
S E
MT
C
ST. THERESE- MTC COLLEGES SEAM 2
Iloilo, Philippines
E

Trim, Stability, Stress


R

C
E

OL
. T H

STUDENT LEARNING MODULE


LEGES
ST

Revision No. 2 Effectivity date: Reviewed by: Approved by:

01 September 2021 QMR President


4
Method 2- Moments of inertia (m ) are tabulated
Tank sounding table for the same tank:
Tank: 2CO.Stbd

Sounding Vol. LCG TCG VCG l


(cms) (cu. M) (m foap) (m) (m) (m4)
0 0.00 78.145 0.063s 1.503 0.0
25 17.57 78.290 2.963s 1.656 364.3
50 41.66 78.345 3.242s 1.785 405.8
75 66.64 78.366 3.365s 1.914 450.4
100 92.49 78.379 3.457s 2.044 497.6
125 118.81 78.389 3.525s 2.174 501.2
150 145.22 78.398 3.569s 2.302 505.0
175 171.69 78.406 3.602s 2.430 508.8
200 198.24 78.414 3.628s 2.557 512.6
225 224.88 78.421 3.648s 2.684 516.4

Moments of Inertia (l) in meters4 are tabulated


The volume of liquid in the tank in cubic meters is tabulated
The density of the liquid is not considered
To obtain the weight (mass) of the liquid and the free surface moments, which are to be
incorporated into the KG moments table, both volume and moments of inertia values as
tabulated must be multiplied by the liquid's density in the tank.

Example:
A ship displaces 5400 t and has a KG of 7.860 m. No. 2 cargo oil tank Stbd. Is filled to a
sounding of 150 cms with cargo oil RD 0.740. Calculate the final effective KG and GM if the
KM for the final condition is 8.000m.
(Use the sounding table extract above)

Solution:
Calculate the mass of oil in the tank;
Mass = Volume x Density; 145.22 x 0.740 = 107.46 t

Calculate the FSMs for oil;


FSMs = l x Density; 505.0 x 0.740 = 373.7 t-m

Calculate the final effective KG or Fluid KG and hence the final effective GM.
Weight (t) KG (m) Moments (t-m)
Ship 5400.00 7.860 42444.0
Cargo oil 107,46 2.302 247.4
FSMs 373.7
Final 5507.46 7.819 43065.1

© All Rights Reserved.


Issue No. 1 Page 102 of 262
S E
MT
C
ST. THERESE- MTC COLLEGES SEAM 2
Iloilo, Philippines
E

Trim, Stability, Stress


R

C
E

OL
. T H

STUDENT LEARNING MODULE


LEGES
ST

Revision No. 2 Effectivity date: Reviewed by: Approved by:

01 September 2021 QMR President


Final Effective KG = Final Moments / Final Weight
= 43065.10 / 5507.46 = 7.819 m
Final Effective KG or Fluid KG

KM 8.000
Fluid KG 7.819
Fluid GM 0.181

Factors Influencing Free Surface Effect:


1. Tank Breadth
If the tank is subdivided, the loss of GM can be significantly reduced.

Example:
A ship has a displacement of 12000 t and an initial KG of 7.84 m. A rectangular double
bottom tank has the following dimensions; length 20 m, breadth 15 m, and filled with
saltwater ballast RD 1.025 to 2 m. If the KM for the final condition is 8.00 m, calculate the
final effective GM.

Solution:
1. Calculate the mass and Kg of the ballast water.
Mass = Volume x Density
= (l x b x sounding) x density
Mass = (20 x 15 x 2) x 1.025 = 615.0 t

Since it is a double bottom tank, the ballast water's Kg will be half the sounding.
Kg = 0.5 x 2.0 = 1.00 m

2. Calculate the FSMs for the rectangular free liquid surface:


FSMs (t-m) = lb3 x dt
12
FSMs (t-m) = 20 x 153 / 12 x 1.025 = 5765.6 t-m

3. Taking moments about the keel calculate the final KG and hence the final GM

Weight (t) KG (m) Moments (t-m)


Initial displ. 12000.00 7.840 94080.0
SW Ballast 615.00 1.000 615.4
FSMs 5765.6
Final 12615.00 7.964 100460.6

Final Effective KG = Final Moments / Final Weight


= 100460.6 / 12615.00 = 7.964 m

© All Rights Reserved.


Issue No. 1 Page 103 of 262
S E
MT
C
ST. THERESE- MTC COLLEGES SEAM 2
Iloilo, Philippines
E

Trim, Stability, Stress


R

C
E

OL
. T H

STUDENT LEARNING MODULE


LEGES
ST

Revision No. 2 Effectivity date: Reviewed by: Approved by:

01 September 2021 QMR President


Final Effective KG or Fluid KG

KM 8.000
Fluid KG 7.964
Fluid GM 0.036

To satisfy the IMO intact stability requirements, the minimum GM requirement for a ship is
0.15 m. This ship does not satisfy that requirement.

Consider the same example, but this time the tank will be equally subdivided into two tanks.

Example:
A ship has a displacement of 12000 t and an initial KG of 7.84 m. A rectangular double
bottom tank has the following dimensions; length 20 m, breadth 15 m, and filled with
saltwater ballast RD 1.025 to 2 m. If the KM for the final condition is 8.00 m, calculate the
final effective GM.
There are two tanks, each having a breadth of 7.5 m.

Solution:
1. Calculate the mass and Kg of the ballast water.
Mass = Volume x Density
= (l x b x sounding) x density
Mass = (20 x 15 x 2) x 1.025 = 615.0 t

Alternatively:
Mass per tank = Volume x Density
= (l x b x sounding) x density
Mass = (20 x 7.5 x 2) x 1.025 = 307.5 t
Total Mass = 307.5 x 2 tank = 615.0 t

Since it is a double bottom tank, the ballast water's Kg will be half the sounding.
Kg = 0.5 x 2.0 = 1.00 m

2. Calculate the FSMs for the rectangular free liquid surface:


FSMs (t-m) = lb3 x dt
12
FSMs (t-m) = 20 x 7.53 / 12 x 1.025 = 720.7 t-m
Total FSMs = 720.7 x 2 tank = 1441.4 t-m

3. Taking moments about the keel calculate the final KG and hence the final GM

Weight (t) KG (m) Moments (t-m)


Initial displ. 12000.00 7.840 94080.0
SW Ballast 615.00 1.000 615.0
FSMs 1441.4
Final 12615.00 7.621 96136.4

© All Rights Reserved.


Issue No. 1 Page 104 of 262
S E
MT
C
ST. THERESE- MTC COLLEGES SEAM 2
Iloilo, Philippines
E

Trim, Stability, Stress


R

C
E

OL
. T H

STUDENT LEARNING MODULE


LEGES
ST

Revision No. 2 Effectivity date: Reviewed by: Approved by:

01 September 2021 QMR President


Final Effective KG = Final Moments / Final Weight
= 96136.4 / 12615.00 = 7.964 m
Final Effective KG or Fluid KG

KM 8.000
Fluid KG 7.621
Fluid GM 0.379

Subdividing the tank has resulted in the final GM being much improved.
This is a direct result of the reduced free surface moments.

Consider the same example, but this time the tank will be equally subdivided into three
tanks.

Example:
A ship has a displacement of 12000 t and an initial KG of 7.84 m. A rectangular double
bottom tank, which is equally subdivided into three compartments, has the following
dimensions; length 20 m, breadth 15 m, and filled with saltwater ballast RD 1.025 sounding
2 m. If the KM for the final condition is 8.00 m, calculate the final effective GM.
There are three tanks, each having a breadth of 5.0 m.

Solution:
1. Calculate the mass and Kg of the ballast water.
Mass = Volume x Density
= (l x b x sounding) x density
Mass = (20 x 15 x 2) x 1.025 = 615.0 t

Alternatively:
Mass per tank = Volume x Density
= (l x b x sounding) x density
Mass = (20 x 5.0 x 2) x 1.025 = 205.0 t
Total Mass = 205.0 x 3 tank = 615.0 t

Since it is a double bottom tank, the ballast water's Kg will be half the sounding.
Kg = 0.5 x 2.0 = 1.00 m

2. Calculate the FSMs for the rectangular free liquid surface:


FSMs (t-m) = lb3 x dt
12
FSMs (t-m) = 20 x 5.03 / 12 x 1.025 = 213.542 t-m
Total FSMs = 213.542 x 3 tank = 640.625 t-m

3. Taking moments about the keel calculate the final KG and hence the final GM

Weight (t) KG (m) Moments (t-m)


Initial displ. 12000.00 7.840 94080.0
© All Rights Reserved.
Issue No. 1 Page 105 of 262
S E
MT
C
ST. THERESE- MTC COLLEGES SEAM 2
Iloilo, Philippines
E

Trim, Stability, Stress


R

C
E

OL
. T H

STUDENT LEARNING MODULE


LEGES
ST

Revision No. 2 Effectivity date: Reviewed by: Approved by:

01 September 2021 QMR President


SW Ballast 615.00 1.000 615.0
FSMs 640.6
Final 12615.00 7.557 95335.6

Final Effective KG = Final Moments / Final Weight


= 95335.6 / 12615.00 = 7.964 m
Final Effective KG or Fluid KG

KM 8.000
Fluid KG 7.557
Fluid GM 0.443

Subdividing the tank into three, it is evident that the final GM is further improved.
This is a direct result of the reduced free surface moments.

Tank Length:
Free surface moments (and loss of GM) are directly proportional to the tank's length, i.e., if
the tank length is double, so will be the value of the free surface moments.

Density:
Free surface moments (and loss of GM) are directly proportional to the liquid's density in the
tank; the more significant the liquid's density in the tank, the greater the FSMs and
subsequent loss of GM.

Ship Displacement:
Free surface moments (and loss of GM) are inversely proportional to the displacement of the
ship. For a given tank, the loss of GM will be smaller as the displacement increases and vice-
versa. It should be noted that the actual free surface moments for any tank are affected by
the ship‟s displacement.

References

3G E-Learning. (2017). Essential seamanship. USA: 3G E-Learning LLC.


3G. E-Learning (2017). Ship and operation management. USA: 3G E-Learning LLC
Patterson, Chris J. & Jonathan D. Ridley. (2014). Ship stability, powering, and
resistance. London: Adlard Coles Nautical.
Arora, Shiven. (2014). Merchant ship naval architecture. New Delhi, India: Venus books.
- Barrass, C.B. and Derrett, D.R. (2012). Ship stability for masters and mates. 7th Ed.
USA: Butterworth-Heinemann.

© All Rights Reserved.


Issue No. 1 Page 106 of 262
S E
MT
C
ST. THERESE- MTC COLLEGES SEAM 2
Iloilo, Philippines
E

Trim, Stability, Stress


R

C
E

OL
. T H

STUDENT LEARNING MODULE


LEGES
ST

Revision No. 2 Effectivity date: Reviewed by: Approved by:

01 September 2021 QMR President

Checkpoint

Activity 1
True or False
1. When the tank is filled with a liquid, the liquid cannot move within the tank when the
ship heels.
2. Any loss in GvM is a loss instability.
3. The increase in KG is affected mainly by the free surface's breadth and is not
dependent upon the tank's mass of liquid.
4. The effect of the free surface is dependent on the position of the tank.
5. Reduction of metacentric height due to free surface effect is more for denser liquid.
6. The lesser transverse surface area of the free surface, the lesser its area moment of
inertia about the tank‟s longitudinal centerline, reduced GM's reduction due to the
free surface effect.
7. The shape of the tank plays a major role in the evaluation of the free surface effect.
8. The ship's stability reduction is directly proportional to the area moment of the free
surface about the tank‟s longitudinal centerline.
9. The effect of the free surface is to increase the sufficient metacentric height from GM
to GvM.
10. Ship heel can cause the ship‟s center of gravity to shift from G to G1, parallel to gg1.

Do this

Activity 2

1. Loading a weight to a position above the vessel‟s center of gravity will cause a:
a. a decrease in KM.
b. shift in the CG away from weight added.
c. an increase in MG
d. shift in the CG towards the weight added.
2. Water on deck will:
a. cause a heel to develop.
b. increase reserve buoyancy.
c. cause FSE.
d. cause LCP.
3. When unloading cargo from the port side lower hold, the center of gravity will move:
a. down and to port.
b. down and to starboard.
c. up and to port.

© All Rights Reserved.


Issue No. 1 Page 107 of 262
S E
MT
C
ST. THERESE- MTC COLLEGES SEAM 2
Iloilo, Philippines
E

Trim, Stability, Stress


R

C
E

OL
. T H

STUDENT LEARNING MODULE


LEGES
ST

Revision No. 2 Effectivity date: Reviewed by: Approved by:

01 September 2021 QMR President


d. up and to starboard.
4. The range of stability of a vessel in a seaway is most affected by:
a. the sum of the free surface.
b. the amount of freeboard.
c. the cargo-carrying capacity.
d. the amount of waterline.
5. In rough weather, the Free Surface Effect resulting from slack tanks in a vessel will
cause a virtual:
a. rise of G
b. rise of K
c. reduction of FSE
d. increase in GM
6. FSE is dependant on the:
a. surface area of a slack tank.
b. length of a slack tank.
c. breadth of a slack tank.
d. depth of a slack tank.
7. To increase the flooding, your vessel could suffer without sinking; you could:
a. raise the center of gravity.
b. increase reserve buoyancy.
c. lower the center of gravity.
d. press up all double bottom tanks.
8. To press up all double bottom tanks and lower the center of gravity of a vessel
suspected of being in a state of loll, you should first fill:
a. the low side of each of the double bottom tanks.
b. both sides of all double bottom tanks simultaneously.
c. the high side of each of the double bottom tanks.
d. the high side of all double bottom tanks simultaneously.
9. When using empty double bottom tanks to lower the center of gravity in a vessel in a
condition of loll, then the tanks to be filled first should be the:
a. longest.
b. narrowest.
c. shortest.
d. widest.
10. The FSE of liquids in tanks can be minimized by:
a. subdividing tanks longitudinally.
b. cutting larger lighting holes in floors.
c. making them wider and longer.
d. subdividing tanks transversely

© All Rights Reserved.


Issue No. 1 Page 108 of 262
S E
MT
C
ST. THERESE- MTC COLLEGES SEAM 2
Iloilo, Philippines
E

Trim, Stability, Stress


R

C
E

OL
. T H

STUDENT LEARNING MODULE


LEGES
ST

Revision No. 2 Effectivity date: Reviewed by: Approved by:

01 September 2021 QMR President

Assessment: Problem Solving

1. M/V St. Therese has a displacement of 13080 t and an initial KG of 7.85 m. A


rectangular double bottom tank has the following dimensions; length 22 m, breadth
15 m, and filled with saltwater ballast RD 1.025 to 2 m. If the KM for the final
condition is 8.10 m, calculate the final effective GM and state if it satisfies the IMO
intact stability requirements.

2. M/T Hide displaces 15050 t and an initial KG of 7.87 m. A rectangular double bottom
tank, which is equally subdivided into three compartments, has a length of 28 m,
breadth of 18 m, and is filled with saltwater ballast RD 1.025, sounding of 2 m. If the
KM for the final condition is 8.00 m, calculate the final effective GM and state if it
satisfies the IMO intact stability requirements.

© All Rights Reserved.


Issue No. 1 Page 109 of 262
S E
MT
C
ST. THERESE- MTC COLLEGES SEAM 2
Iloilo, Philippines
E

Trim, Stability, Stress


R

C
E

OL
. T H

STUDENT LEARNING MODULE


LEGES
ST

Revision No. 2 Effectivity date: Reviewed by: Approved by:

01 September 2021 QMR President

Lesson 10

Learning Module 10.1 Trim & draught calculation using trim tables
(5 hours)

Competence, Course Outcomes, and Learning Outcomes

Competence:

A-II/1 F3.C2: Maintain seaworthiness of the ship

Course Outcome/s:

Calculate ship stability in compliance with the IMO intact stability


criteria under all conditions of loading

Learning Outcomes:

At the end of learning module, the student can:

1. Calculate trim and draught.

Overview

On this section will discuss the calculation of ship stability in compliance with the
IMO criteria under all conditions of loading in relation to the trim and draught.
Factors concerning computation, formulas and process of solving problems, correct
reading of draft, point of measurement, terms and definition concerning on the
topics. Sample solving problems of trim calculations and check point are provided for
student activity.

© All Rights Reserved.


Issue No. 1 Page 110 of 262
S E
MT
C
ST. THERESE- MTC COLLEGES SEAM 2
Iloilo, Philippines
E

Trim, Stability, Stress


R

C
E

OL
. T H

STUDENT LEARNING MODULE


LEGES
ST

Revision No. 2 Effectivity date: Reviewed by: Approved by:

01 September 2021 QMR President


Vessel Trim Calculation
Trim is different between the aft draft
and forward draft of the vessel.
Trim = Draft Aft (DA) – Draft Fore
(DF)

Example: DF: 5.40 M, and DA: 7.25 M


Trim = 7.45 – 5.40
Trim = 2.05 M (Apparent Trim / AT)

Draft Corrected to Perpendicular

The After Perpendicular is a right-angle line to the keel passing through the rudder post; it is
also the first frame marked on the vessel‟s drawings.
The Forward Perpendicular is a right-angle line to the keel, cutting the vessel‟s Summer
waterline at the stem. The vessel's stability information is calculated on the drafts measured
at the perpendiculars; as the draft marks rarely coincide with these lines, a draft as read
must be corrected. If the marks are not on the perpendiculars, the vessel usually has a
tabulated plan in her hydrostatic books.

However, some of the older vessels do not have their tabulation. Therefore, it is necessary
to work out the correction to be applied by referring to the vessel's capacity plan and
measuring the horizontal distance between the draft marks and the waterline's
perpendiculars.
Forward Correction = (AT x Fd)/(LBM)
Aft Correction = (AT x Ad)/(LBM)
Midships Correction = (AT x Md)/(LBM)
Fd: distance Draft Forward to forwarding Perpendiculars
Md: distance Draft midship to midship Fd)

© All Rights Reserved.


Issue No. 1 Page 111 of 262
S E
MT
C
ST. THERESE- MTC COLLEGES SEAM 2
Iloilo, Philippines
E

Trim, Stability, Stress


R

C
E

OL
. T H

STUDENT LEARNING MODULE


LEGES
ST

Revision No. 2 Effectivity date: Reviewed by: Approved by:

01 September 2021 QMR President

Discussion

Key ConceptForward Perpendicular – the vertical line of reference that intersects the
summer load waterline at the stem's forward edge when the ship is on an even keel.
After Perpendicular – the vertical line of reference coincides with the after edge of the
sternpost, or if no sternpost, then the rudder's turning axis.
Length between Perpendicular – is the horizontal distance between the forward and
after perpendiculars. It is this length that is considered when conducting trim calculations.
Length Overall – is the horizontal distance between the aftermost part and forward-most
part of the ship.
Amidships – the mid-point between the forward and aft perpendicular. It is not the mid-
point in the length of the ship.

Draught Marks and Reading the Draught


The draught marks on a ship should be marked at the forward and aft perpendiculars
on both sides. The numerals are 10 cm. in height and are spaced 10 cm. apart.

© All Rights Reserved.


Issue No. 1 Page 112 of 262
S E
MT
C
ST. THERESE- MTC COLLEGES SEAM 2
Iloilo, Philippines
E

Trim, Stability, Stress


R

C
E

OL
. T H

STUDENT LEARNING MODULE


LEGES
ST

Revision No. 2 Effectivity date: Reviewed by: Approved by:

01 September 2021 QMR President


Read the draught for each of the waterlines in meters.

Vessel draft or vessel draught

Vessel draft is one of the principal dimensions of any waterborne vessel and is defined in
technical terms as the distance between the ship‟s keel and the vessel's waterline.

The keel of a ship is the lowest longitudinal support member that helps form the vessel's
structure. At the same time, the waterline of any floating body is the location on the hull of
a ship where the air-water interface occurs.

Trim:
Trim is the difference in centimeters or meter between the forward and aft draughts,
as measured at the forward and aft perpendiculars.
Draughts Fwd-2.20m and Aft 2.68m

Draughts Fwd-2.20m - Aft 2.68m = 0.48m or 48cms by the stern


Draughts Fwd-2.70m and Aft 2.32m
© All Rights Reserved.
Issue No. 1 Page 113 of 262
S E
MT
C
ST. THERESE- MTC COLLEGES SEAM 2
Iloilo, Philippines
E

Trim, Stability, Stress


R

C
E

OL
. T H

STUDENT LEARNING MODULE


LEGES
ST

Revision No. 2 Effectivity date: Reviewed by: Approved by:

01 September 2021 QMR President

Draughts Fwd-2.70m - Aft 2.32m = 0.38m or 38cms by the head

Change of Trim:
If weight is shifted longitudinally, the ship will experience a change of trim.

Example:
A ship floats at draught Fwd 6.000 m and Aft 6.200 m. a weight is then shifted forward. The
final draught is Fwd 6.080 m and Aft 6.120 m.
What change of trim has occurred?
Solution:
Calculate the initial trim:
Aft 6.200 m
Fwd 6.000 m
Initial trim 0.200 m by the stern

Calculate the final trim:


Aft 6.120 m
Fwd 6.080 m
Final trim 0.040 m by the stern

Calculate the change of trim (COT):


Initial trim 0.200 m by the stern
Final trim 0.040 m by the stern
COT 0.160 m by the head

The trim by the stern has been reduced


by 0.160 m.

Example:
A ship floats at draught Fwd 5.000 m and Aft 4.640 m. a weight is then shifted aft. The final
draught is Fwd 4.680 m and Aft 4.960 m. what change of trim has occurred?
Solution:
Calculate the initial trim:
Aft 4.640 m
Fwd 5.000 m
Initial trim 0.360 m by the head

Calculate the final trim:


Aft 4.960 m

© All Rights Reserved.


Issue No. 1 Page 114 of 262
S E
MT
C
ST. THERESE- MTC COLLEGES SEAM 2
Iloilo, Philippines
E

Trim, Stability, Stress


R

C
E

OL
. T H

STUDENT LEARNING MODULE


LEGES
ST

Revision No. 2 Effectivity date: Reviewed by: Approved by:

01 September 2021 QMR President


Fwd 4.680 m
Final trim 0.280 m by the stern

Calculate the change of trim:


Initial trim 0.360 m by the head
Final trim 0.280 m by the stern
COT 0.640 m by the stern

The ship, initially trimmed by the head, completes trimmed by the stern.

Moment to change trim by one centimeter (MCTC)

This is the trimming moment required to change the ship's trim by exactly one centimeter.
It is tabulated in the ship hydrostatic particulars and is used to determine the trim change
when weights are shifted, loaded, or discharged.

Consider the ship showed figure 12.7-12.9, where weight was shifted aft along the deck.
The formula can calculate the change of trim:
COT (cms) = Trimming moment
MCTC
Where the trimming moment is: w x d
„w‟ is the weight shifted, and
„d‟ is the distance through which the weight is shifted longitudinally.
Thus COT (cms) = w x d
MCTC
Example:
A weight of 150 tonnes is moved aft by a distance of 20 m. if the MCTC for the current
draught is 250 m-t, determine the ship's final
trim if the initial trim was 0.20 m by the stern.
Solution:
COT (cms) = w x d = 150 x 20 = 12 cms
MCTC 250
COT = 0.120 m
Initial trim: 0.200 m by the stern
COT: 0.120 m further by the stern
Final trim: 0.320 m by the STERN

The formula for Calculating MCTC


In practice, the MCTC value will always be found for the draught in question in the ship‟s
hydrostatic particulars. However, in examinations, it may have to be calculated, and the
formula for calculating MCTC is:

MCTC = W x GML
100LBP
Where:
„W‟ is the ship‟s displacement;
„GML‟ is the longitudinal metacentric, &;

© All Rights Reserved.


Issue No. 1 Page 115 of 262
S E
MT
C
ST. THERESE- MTC COLLEGES SEAM 2
Iloilo, Philippines
E

Trim, Stability, Stress


R

C
E

OL
. T H

STUDENT LEARNING MODULE


LEGES
ST

Revision No. 2 Effectivity date: Reviewed by: Approved by:

01 September 2021 QMR President


„LBP‟ is the length between perpendiculars.

The derivation of this formula is as follows.


The ship in Figure 12.10 is on an even keel with a weight on deck.
The weight is shifted aft along the deck through distance „d‟ meters. By the formula:

GG1 = w x d
W
G will move aft to G1 (parallel to and in the same direction as the weight shift).
G and B become horizontally separated, creating a trimming lever. This causes the ship to
trim by the stern until B attains a new position vertically below the new longitudinal center
of gravity, G1 (Figure 12.12)

GG1 ML is a right-angled triangle where:


Tan ø = OPP
ADJ

Therefore: Tan ø = GG1


GML

Since: GG1 = w x d
W

And; Tan ø = GG1


GML

Then: Tan ø = w x d
W x GML

Also, in Figure 12.13: Tan ø = Trim(m)


LBP(m)

Tan ø = 0.01 (m) which equals: Tan ø = 1


LBP (m) 100 LBP

Tan ø = w x d equals: Tan ø = MCTC


W x GML W x GML

1 = MCTC
100LBP W x GML

It gives: MCTC = W x GML


100LBP
Longitudinal Center of Flotation (LCF or F)
This is at the geometric center of the ship‟s waterplane area and is the point about
which the ship will trim. The position of the LCF is important because if a ship experiences a

© All Rights Reserved.


Issue No. 1 Page 116 of 262
S E
MT
C
ST. THERESE- MTC COLLEGES SEAM 2
Iloilo, Philippines
E

Trim, Stability, Stress


R

C
E

OL
. T H

STUDENT LEARNING MODULE


LEGES
ST

Revision No. 2 Effectivity date: Reviewed by: Approved by:

01 September 2021 QMR President


change of trim, some of that change of trim must be applied to the aft draught and the
remainder applied to the forward draught, as can be seen.
Aft draught increases, Forward draught decreases.

Calculating the final draught when weight is shifted


The position of the LCF determines how the change of (COT) will be apportioned between
the forward and aft draughts.

Ship with LCF amidship

Consider a ship with weight on deck. The weight is


shifted forward, causing the ship to change trim.
COT (cms) = W x d
MCTC
The change of trim has to be shared between the
forward and aft draught.
Ta – is a change in draught aft due to trim
Tf – is the change in draught forward due to trim
If the LCF is amidships, the change of trim (COT) will be
apportioned equally to the draught forward and aft, Ta = COT / 2 and Tf = COT / 2

Example:
A ship floats at draught F 6.50m and A 6.80m. Determine the final draught if 25 tons is
moved 45 m forward given that MCTC is 112.5 t- m and the LCF are amidship.

Solution:

Calculate the change of trim (COT)


COT = w x d = 25 x 45 = 10 cms by the head
MCTC 112.5

Apportion the COT equally to the forward and aft draughts

Ta = 10 = 5 cms (Aft draught will reduce by 5 cms)


2

Tf = 10 = 5 cms (forward draught will increase by 5 cms)


2

Ta = -0.050 m Tf = +0.050 m

Calculate the final draught

Initial draught F 6.500 A 6.800


Trim +0.050 -0.050

© All Rights Reserved.


Issue No. 1 Page 117 of 262
S E
MT
C
ST. THERESE- MTC COLLEGES SEAM 2
Iloilo, Philippines
E

Trim, Stability, Stress


R

C
E

OL
. T H

STUDENT LEARNING MODULE


LEGES
ST

Revision No. 2 Effectivity date: Reviewed by: Approved by:

01 September 2021 QMR President


Final F 6.550 m A 6.750 m

Ship with LCF, not Amidships

Consider a ship with a weight on deck. The LCF is aft of amidship.


The weight is shifted forward, causing the ship to change trim by the head where:

COT (cms) = w x d
MCTC

In this case, it can be seen that the changes in draught forward and aft are not the same.

The trim (COT) change will have to be apportioned to the forward and aft draught according
to the position of the LCF within the ship‟s length.
„a‟ is the length of shift aft of the LCF.
„f‟ is the length of the ship forward of the LCF.
COT = Ta + Tf

Consider the similar triangles where:

Tan ø = OPP
ADJ
Tan ø = Tf and; Tan ø = COT
f LBP
therefore: Tf = COT
f LBP
Rearranging this gives: Tf = f x COT and it follows that: Ta = a x COT
LBP LBP

Example
A ship has initial draught F 10.25 m and 10.15 m. a weight of 95 tonnes is moved aft
through a distance of 42 m. calculate the final draughts given that LBP is 100 m, LCF is 48
m foap, and MCTC is 285 t-m.

Solution:

Calculate the change of trim (COT)


COT = w x d = 95 x 42 = 14 cms by the stern
MCTC 285

Apportion the COT according to the LCF position


Ta = 48 x 14 = 6.7 cms Tf = 52 x 14 = 7.3 cms
100 100
100
(Alternatively: Tf = COT - Ta; Tf = 14.0 – 6.7 = 7.3 cms)

© All Rights Reserved.


Issue No. 1 Page 118 of 262
S E
MT
C
ST. THERESE- MTC COLLEGES SEAM 2
Iloilo, Philippines
E

Trim, Stability, Stress


R

C
E

OL
. T H

STUDENT LEARNING MODULE


LEGES
ST

Revision No. 2 Effectivity date: Reviewed by: Approved by:

01 September 2021 QMR President

Weight is moved aft so the ship will trim by the STERN.


Ta = +0.067 m Tf = -0.073 m

Calculate the final draughts


Initial draught F 10.250 A 6.800
Trim -0.073 +0.067
Final F 10.177 m A 10.217 m

The Effect of Loading and Discharging Weights


If the weights are loaded or discharged, the effect of bodily Sinkage or rise must also be
considered.
Sinkage/Rise cms = w
TPC

The following procedure should be followed when loading a weight:


1. Load the weight at the LCF position and calculate the Sinkage using the TPC value
given.
(If weight is loaded at the LCF, the ship will sink uniformly, there will be no trim
change!)
2. Calculate the COT by moving the weight from the LCF position to its actual loaded
position.
3. Find Ta/Tf by apportioning the COT according to the position of the LCF.
4. Apply both the Sinkage and Ta/Tf to the initial draughts to determine the final
draughts.

Example:
A ship 100 m in length floats at draught F 7.00 m and A 6.80 m. Calculate the final draughts
if 150 t is loaded 20 m foap given that TPC is 15 and MCTC is 150 t-m, and LCF is 45 m
foap.

TIP: Always sketch to help you picture what is happening

Solution:
Calculate the sinkage Sinkage = w = 150 = 10 cms = 0.100 m

© All Rights Reserved.


Issue No. 1 Page 119 of 262
S E
MT
C
ST. THERESE- MTC COLLEGES SEAM 2
Iloilo, Philippines
E

Trim, Stability, Stress


R

C
E

OL
. T H

STUDENT LEARNING MODULE


LEGES
ST

Revision No. 2 Effectivity date: Reviewed by: Approved by:

01 September 2021 QMR President


TPC 15
Calculate the change of trim (COT)
(„d‟ is the distance that the weight is loaded from the LCF)
COT = w x d = 150 x (45-20) = 25.0 cms
MCTC 150
Ta = 45 x 25.0 = 11.3 cms = 0.113 m
100
Tf = COT – Ta; Tf = 25.0 – 11.3 = 13.7 cms = 0.137 m
Weight is loaded aft of the LCF so that the ship will trim by the STERN.
Calculate the final draughts

Initial draughts F 7.000 A 6.800


Sinkage +0.100 +0.100
7.100 6.900
Trim -0.137 +0.113
Final F 6.963 m A 7.013 m

The same procedures apply when discharging a weight:


1. Discharge the weight from the LCF position and calculate the ship's rise using the
TPC value given.
2. Calculate the COT by considering the effect of moving the weight from its original
position to the LCF.
3. Find the Ta/Tf by apportioning the COT according to the position of the LCF.
4. Apply both the rise and Ta/Tf to the initial draughts to determine the final draughts.

Example:
A with LBP 160 m floats at draughts F 3.22 m and A 3.10 m. Calculate the final draughts if
208 t is discharged from a 118 m foap given that TPC is 32, MCTC is 306 t-m, and LCF is 88
m foap.

Solution:
Calculate the rise
Rise = w = 208 = 6.5 cms = 0.065 m
TPC 32
Calculate the change of trim (COT)

© All Rights Reserved.


Issue No. 1 Page 120 of 262
S E
MT
C
ST. THERESE- MTC COLLEGES SEAM 2
Iloilo, Philippines
E

Trim, Stability, Stress


R

C
E

OL
. T H

STUDENT LEARNING MODULE


LEGES
ST

Revision No. 2 Effectivity date: Reviewed by: Approved by:

01 September 2021 QMR President


(„d‟ is the distance that the weight is discharged from the LCF)
COT = w x d = 208 x (118-88) = 20.4 cms
MCTC 306
Ta = 88 = 20.4 = 11.2 cms = 0.112 m
160
Tf = COT – Ta = 20.4 – 11.2 = 9.2 cms = 0.092 m
Weight is discharged from forward of the LCF so that the ship will trim by the STERN.

Calculate the final draughts


Initial draughts F 3.220 A 3.100
Rise -0.065 -0.065
3.155 3.035
Trim -0.092 +0.112
Final F 3.063 m A 3.147 m

Multiple Weight Problems


A problem involving multiple weights requires a tabular approach to be adopted where
moments are taken about the LCF.

Example:
A ship 120 m in length floats at draughts F 6.24 m and A 6.36 m. LCF is 54 m foap, TPC
14.2 and MCTC 116 t-m.
The following cargo is worked:
Load 120 t lcg 10.0 m foap;
Load 68 t lcg 86 m foap;
Discharged 36 t lcg 22 m foap;
Discharged 48 t lcg 60 m foap.
Calculate the final draughts

Solution
In column one (1), the weights loaded and discharged are summed to find the net weight
loaded or discharged.
In column two (2), the weights are listed as positive values, regardless of whether they are
being loaded or discharged.
In column three (3), the distance that each weight is loaded or discharged from the LCF is
listed.
Trimming moments are calculated (column 2 being multiplied by column 3) and entered in
column (4) or (5) depending on whether they are head or stern moments.
Be careful to apply the correct way.

Determine the net trimming moments – in this case, 2240 t-m by the stern.
1 2 3 4 5
Weight (t) Weight (t) Dist. From Head moments Stern moments
LCF (m) (t-m) (t-m)
120 120 44 5280
68 68 32 2176

© All Rights Reserved.


Issue No. 1 Page 121 of 262
S E
MT
C
ST. THERESE- MTC COLLEGES SEAM 2
Iloilo, Philippines
E

Trim, Stability, Stress


R

C
E

OL
. T H

STUDENT LEARNING MODULE


LEGES
ST

Revision No. 2 Effectivity date: Reviewed by: Approved by:

01 September 2021 QMR President


-36 36 32 1152
-48 48 6 288
104 load 3328 5568
3328
2240

Calculate the Sinkage/rise of the ship

Sinkage (cms) = w / TPC = 104 / 14.2 =7.3 cms = 0.073 m

Calculate the change of trim

COT = Trimming moments / MCTC = 2240 / 116 = 19.3 cms by the stern

Ta = 54 / 120 x 19.3 = 8.7 cms = 0.087 m

Tf = COT – Ta = 19.3 – 8.7 = 10.6 cms = 0.106 m

Calculate the final draught


Initial draughts F 6.240 A 3.100
Sinkage +0.073 +0.073
6.313 6.433
Trim -0.106 +0.087
Final F 6.207 m A 6.520 m

USEFUL COMMON FORMULAE:


Sinkage/rise(cms) = w/TPC
COT (cms) = wxd/MCTC = Trimming moments/MCTC
Ta = a/LBP x COT Tf = COT – Ta

TRIM may be considered as the longitudinal equivalent of the list. Trim is also known as
„longitudinal stability.‟
Instead of trim being measured in degrees, it is measured as the difference between the
drafts forward and aft.
If the difference is zero, then the ship is on an even keel.
If the forward draft is more greater than the aft draft, it is trimmed by the bow.
If the aft draft is greater than the forward draft, the vessel is trimmed by the stern.

Causes of Trim:
1. Moving loads fore and aft from one point to another changes the ship's trim but not
the mean draft.
2. Adding or removing loads to and from the ship will involve a decrease or increase of
the mean draft and a change in trim.
3. The change in water density where the ship is floating changes the mean draft and
the trim of the ship.

© All Rights Reserved.


Issue No. 1 Page 122 of 262
S E
MT
C
ST. THERESE- MTC COLLEGES SEAM 2
Iloilo, Philippines
E

Trim, Stability, Stress


R

C
E

OL
. T H

STUDENT LEARNING MODULE


LEGES
ST

Revision No. 2 Effectivity date: Reviewed by: Approved by:

01 September 2021 QMR President

References

– 3G E-Learning. (2017). Essential seamanship. USA: 3G E-Learning LLC.


– 3G. E-Learning (2017). Ship and operation‟s management. USA: 3G E
Learning LLC
- Patterson, Chris J. & Jonathan D. Ridley. (2014). Ship stability, powering and
resistance. London: Adlard Coles Nautical.
- Arora, Shiven. (2014). Merchant ship naval architecture. New Delhi, India:
Venus books.
- Barrass, C.B. and Derrett, D.R. (2012). Ship stability for masters and mates. 7th Ed.
USA: Butterworth-Heinemann.
- Rhodes, Martin A. (BSc. Hons) (2013). Ship Stability for Mates/Masters. 4 Dunlop
Square, Livingston Edinburgh, EH54 8SB Scotland, UK

© All Rights Reserved.


Issue No. 1 Page 123 of 262
S E
MT
C
ST. THERESE- MTC COLLEGES SEAM 2
Iloilo, Philippines
E

Trim, Stability, Stress


R

C
E

OL
. T H

STUDENT LEARNING MODULE


LEGES
ST

Revision No. 2 Effectivity date: Reviewed by: Approved by:

01 September 2021 QMR President

Checkpoint

Activity 1

A ship 90m long is floating at drafts 4.5m F and 5.0m A. The Centre of flotation is
1.5m aft of amidships. TPC 10 tonnes. MCTC 120 tons m. Find the new drafts if a
weight of 450 tons already onboard is shifted in a position 14m forward of
amidships.
Show your complete solution.

Do this

Activity 2

Fill in the blanks.

1. _____ may be considered as the longitudinal equivalent of the list.


2. If the difference between the drafts forward and aft is zero, then the ship is on
_____.
3. If the forward draft is greater than the aft draft, the vessel is trimmed by the _____.
4. If the aft draft is greater than the forward draft, the vessel is trimmed by the _____.
5. The change in water density where the ship is floating changes the ____ as well as
the trim of the ship.

© All Rights Reserved.


Issue No. 1 Page 124 of 262
S E
MT
C
ST. THERESE- MTC COLLEGES SEAM 2
Iloilo, Philippines
E

Trim, Stability, Stress


R

C
E

OL
. T H

STUDENT LEARNING MODULE


LEGES
ST

Revision No. 2 Effectivity date: Reviewed by: Approved by:

01 September 2021 QMR President

Assessment

1. A ship 100m long and a displacement of 2,200 tons has a longitudinal metacentric height
of 150m. The present drafts are 5.2m F, and 5.3m A. Centre of flotation is 3m aft of
amidships. Find the new drafts if a weight of 5 tons already onboard is shifted aft
through a distance of 60m of fp.
Hint: Review the formula for finding MCTC.
Show your complete solution.
Calculate MCTC = W x GML /100LBP = 2200 t x 150 m / 100 x 100 = 33 t-m
Calculate the change of trim (COT)
COT = w x d = 5 x 7 = 1.06 cms by the stern
MCTC 33

Apportion the COT according to the LCF position


Ta = 47 x 1.06 = 0.50 cms Tf = 53 x 1.06 = 0.56 cms
100 100

Weight is moved aft so that the ship will trim by the aft.
Ta = +0.005 m Tf = -0.006 m

Calculate the final draughts


Initial draught F 5.200 A 5.300
Trim -0.006 +0.005
Final draught F 5.194 m A 5.305m

© All Rights Reserved.


Issue No. 1 Page 125 of 262
S E
MT
C
ST. THERESE- MTC COLLEGES SEAM 2
Iloilo, Philippines
E

Trim, Stability, Stress


R

C
E

OL
. T H

STUDENT LEARNING MODULE


LEGES
ST

Revision No. 2 Effectivity date: Reviewed by: Approved by:

01 September 2021 QMR President

Lesson 11

Learning Module 11.1 Actions to be taken in the event of partial


loss of intact buoyancy (2.5 hours)

Competence, Course Outcomes and Learning Outcomes

Competence:

A-II/1 F3.C2: Maintain seaworthiness of the ship

Course Outcome/s:

Calculate ship stability in compliance with the IMO intact stability


criteria under all conditions of loading

Learning Outcomes:

At the end of the learning module , the student can:

2. Determine the actions to take during flooding.

Overview

Compartments that are only partially flooded because their outboard bulkheads contain
small holes, cracks, loose rivets, broken seams, or splinter holes, allow progressive
flooding to take place. If nothing is done about these holes, the ship will lose buoyancy
and list or trim stability. Partial flooding refers to a condition On this sections will discuss
the different conditions of compartment on every different types of ship according to
their cargo loaded, and action to be taken in the event of damage compartment with the
danger of flooding.

© All Rights Reserved.


Issue No. 1 Page 126 of 262
S E
MT
C
ST. THERESE- MTC COLLEGES SEAM 2
Iloilo, Philippines
E

Trim, Stability, Stress


R

C
E

OL
. T H

STUDENT LEARNING MODULE


LEGES
ST

Revision No. 2 Effectivity date: Reviewed by: Approved by:

01 September 2021 QMR President

Discussion

Key Concept: ACTIONS TO BE TAKEN IN THE EVENT OF PARTIAL LOSS OF


INTACT BUOYANCY

Actions to be Taken in the Event of Partial Loss of Intact Buoyancy

• The immediate actions which should be taken by the officer in charge of the watch
are aimed at limiting the volume of lost buoyancy to the minimum.
• At the same time, if cross-flooding arrangements are required, they should be put
into operation immediately to restrict the angle of list.
• Whether anything can be done to stop or reduce the inflow of water will depend
upon the circumstances.
• In the event of loss of buoyancy due to damage to a hatch cover, a prompt reduction
in speed or alteration of course, or both, may be effective.

Damage Stability
• A damage stability criterion varies from ship to ship and the requirement for the
same is given in SOLAS chapter II-1. It may be single compartment flooding, multi
compartment flooding, engine room flooding etc.
• Under all the criteria as applicable, vessel margin line should not be submerged after
the damage. Margin line is an imaginary line drawn 75mm below the free board
deck.
• In order to ensure sufficient floatability and stability after damage it is vital to
prevent water propagating further through the buoyant parts of the ship.
• The volume of tanks and spaces must be limited with watertight bulkheads to
improve stability
© All Rights Reserved.
Issue No. 1 Page 127 of 262
S E
MT
C
ST. THERESE- MTC COLLEGES SEAM 2
Iloilo, Philippines
E

Trim, Stability, Stress


R

C
E

OL
. T H

STUDENT LEARNING MODULE


LEGES
ST

Revision No. 2 Effectivity date: Reviewed by: Approved by:

01 September 2021 QMR President


• Keeping watertight doors closed must be vital to survival.
• Internationally, more radical means of maintaining buoyancy have been proposed,
such as the use of high-density foam in double hulled compartments.
• Greater subdivision, the duplication of all ship systems to ensure that they are not
knocked out in a collision, grounding or fire are more conventional approaches that
will tend to be employed to make a loss of buoyancy less likely in an accident.

Ships Types
 General cargo
 Oil, chemical and gas tankers
 Container ships
 Bulk carrier
 Roll-on/roll-off (RORO or ro-ro)
 Passenger ship

1. General Cargo Ships

General cargo - In shipping, break bulk cargo or general cargo are goods that
must be loaded individually, and not in intermodal containers nor in bulk as with oil
or grain. Ships that carry this sort of cargo are often called general cargo ships.

© All Rights Reserved.


Issue No. 1 Page 128 of 262
S E
MT
C
ST. THERESE- MTC COLLEGES SEAM 2
Iloilo, Philippines
E

Trim, Stability, Stress


R

C
E

OL
. T H

STUDENT LEARNING MODULE


LEGES
ST

Revision No. 2 Effectivity date: Reviewed by: Approved by:

01 September 2021 QMR President

2. Oil, chemical and gas tankers

© All Rights Reserved.


Issue No. 1 Page 129 of 262
S E
MT
C
ST. THERESE- MTC COLLEGES SEAM 2
Iloilo, Philippines
E

Trim, Stability, Stress


R

C
E

OL
. T H

STUDENT LEARNING MODULE


LEGES
ST

Revision No. 2 Effectivity date: Reviewed by: Approved by:

01 September 2021 QMR President

3. Passenger ships

4. Container ships

© All Rights Reserved.


Issue No. 1 Page 130 of 262
S E
MT
C
ST. THERESE- MTC COLLEGES SEAM 2
Iloilo, Philippines
E

Trim, Stability, Stress


R

C
E

OL
. T H

STUDENT LEARNING MODULE


LEGES
ST

Revision No. 2 Effectivity date: Reviewed by: Approved by:

01 September 2021 QMR President


5. Bulk carriers

6. Roll-on/roll-off (RORO or ro-ro)

© All Rights Reserved.


Issue No. 1 Page 131 of 262
S E
MT
C
ST. THERESE- MTC COLLEGES SEAM 2
Iloilo, Philippines
E

Trim, Stability, Stress


R

C
E

OL
. T H

STUDENT LEARNING MODULE


LEGES
ST

Revision No. 2 Effectivity date: Reviewed by: Approved by:

01 September 2021 QMR President

References

– 3G E-Learning. (2017). Essential seamanship. USA: 3G E-Learning LLC.


– 3G. E-Learning (2017). Ship and operation‟s management. USA: 3G E
Learning LLC
- Patterson, Chris J. & Jonathan D. Ridley. (2014). Ship stability, powering and
resistance. London: Adlard Coles Nautical.
- Arora, Shiven. (2014). Merchant ship naval architecture. New Delhi, India:
Venus books.
- Barras, C.B. and Darrett, D.R. (2012). Ship stability for masters and mates. 7th Ed.
USA: Butterworth-Heinemann.

Activity 1

Identification

1. Ships that carry goods in bulk.


2. Ships that the primary function is to carry passengers.
3. Merchant ships designs for bulk transfer of oil.
4. A cargo ship that carries all of their load in truck-size intermodal containers.
5. Ships design to carry wheeled cargo.

Write TRUE if the sentence is correct, and write the CORRECT ANSWER if the
underline word does not satisfy the sentence.

1. Bulk carrier is design to transport unpacked bulk cargo.


2. Margin line is an imaginary line drawn 75mm below the free board deck.
3. Ships that carry automobiles and trucks is container ship.
4. The two basic types of oil tankers are crude tankers and product tankers.
5. Product tankers move large quantities of crude oil from its point of extraction to
refineries.

© All Rights Reserved.


Issue No. 1 Page 132 of 262
S E
MT
C
ST. THERESE- MTC COLLEGES SEAM 2
Iloilo, Philippines
E

Trim, Stability, Stress


R

C
E

OL
. T H

STUDENT LEARNING MODULE


LEGES
ST

Revision No. 2 Effectivity date: Reviewed by: Approved by:

01 September 2021 QMR President

Assessment

EXPLAIN:

ACTIONS TO BE TAKEN IN THE EVENT OF PARTIAL LOSS OF INTACT BUOYANCY

We should take immediate actions which, especially should be taken by the officer in
charge of the watch and are aimed at limiting the volume of lost buoyancy to the minimum.
At the same time, if cross-flooding arrangements are required, we should be put into
operation immediately to restrict the angle of list. Whether anything can be done to stop or
reduce the inflow of water will depend upon the circumstances. If nothing is done about
these holes, the ship will lose buoyancy and list or trim stability. If the boundaries remain
intact, water will neither run into nor out of the flooded compartment as the ship rolls. The
final result of partial flooding is usually a decided loss in overall stability. In the event of loss
of buoyancy due to damage to a hatch cover, a prompt reduction in speed or alteration of
course, or both, may be effective.

© All Rights Reserved.


Issue No. 1 Page 133 of 262
S E
MT
C
ST. THERESE- MTC COLLEGES SEAM 2
Iloilo, Philippines
E

Trim, Stability, Stress


R

C
E

OL
. T H

STUDENT LEARNING MODULE


LEGES
ST

Revision No. 2 Effectivity date: Reviewed by: Approved by:

01 September 2021 QMR President

Lesson 11

Learning Module 11.2 Stress tables & stress calculating


Equipment (2.5 hours)

Competence, Course Outcomes and Learning Outcomes

Competence:

A-II/1 F3.C2: Maintain seaworthiness of the ship

Course Outcome/s:

Calculate ship stability in compliance with the IMO intact stability


criteria under all conditions of loading

Learning Outcomes:

At the end of learning module, the student can:

.1. Determine stress in ships using stress table and stress -calculating equipment.

Overview

Stress on Ships

The modern ship is made up steel plating, section and builds up girders so connected as
to provide adequate strength in all parts to withstand the forces acting on the ship under all
condition of service.
The forces acting on a ship may be static or dynamic. The static forces are due to
the difference in the weight and buoyancy, which occur throughout the ship. The dynamic
forces are cause by the motion of the ship at sea and the action of the wind and wave.
On this lesson will discuss the different type of stresses, dynamic forces acting on the ship ,
on the last part students will perform checkpoints.

© All Rights Reserved.


Issue No. 1 Page 134 of 262
S E
MT
C
ST. THERESE- MTC COLLEGES SEAM 2
Iloilo, Philippines
E

Trim, Stability, Stress


R

C
E

OL
. T H

STUDENT LEARNING MODULE


LEGES
ST

Revision No. 2 Effectivity date: Reviewed by: Approved by:

01 September 2021 QMR President

Discussion

Key Concept

These forces create:

1. Longitudinal stress
2. Transverse stress
3. Local stress

Longitudinal Stress

Hogging and sagging can be tolerated, but only within limits. When they exceed acceptable
limits, hogging or sagging will cause the hull to crack in the area of greatest stress.

In an improper loading of a vessel, a hogged or sagged condition can be caused by a


trochoidal wave action, an action of a wave whose distance between crests is equal to the
length of the ship and whose height is 1/20 of its length.

When a vessel encounters a wave of that size, it will be hogged when the midsection of the
vessel passes over a crest and sagged when the bow and stern sections are supported by
crests. In determining whether a vessel is hogged or sagged it is helpful to clear mental
uncertainties if look at draft as a "theoretical" draft and load (midships) draft as an "actual"
draft.

© All Rights Reserved.


Issue No. 1 Page 135 of 262
S E
MT
C
ST. THERESE- MTC COLLEGES SEAM 2
Iloilo, Philippines
E

Trim, Stability, Stress


R

C
E

OL
. T H

STUDENT LEARNING MODULE


LEGES
ST

Revision No. 2 Effectivity date: Reviewed by: Approved by:

01 September 2021 QMR President

HOGGING

Hogging - is the straining of a ship lightly loaded amidships and more heavily
forward and aft, causing the bow and stern to be lower than the middle section. Consider a
ship loaded with the weights concentrated at the bow and the stern, which tends to droop.
This leads to hogging of the ship hull.

SAGGING

 If the weight amidships exceeds the buoyancy or when the wave trough amidships the
ship will sag, as a beam supported at an end and loaded at mid length.

© All Rights Reserved.


Issue No. 1 Page 136 of 262
S E
MT
C
ST. THERESE- MTC COLLEGES SEAM 2
Iloilo, Philippines
E

Trim, Stability, Stress


R

C
E

OL
. T H

STUDENT LEARNING MODULE


LEGES
ST

Revision No. 2 Effectivity date: Reviewed by: Approved by:

01 September 2021 QMR President


Sagging due to discontinuity in loading

Sagging - is the straining of a ship heavily loaded amidships and more lightly
forward and aft, causing the bow and stern to be higher than the middle section. Consider
heavy weights concentrated at the midships of a ship. The middle hull part tends to drop
more than the ends. This causes sagging of ship hull.

Transverse Stress

 A transverse section of amidships is subjected to static pressure due to the


surrounding water as well as internal loading due to the weight of the structure, cargo,
etc.

The parts of the structure, which resist transverses, are

1.Transverse bulkhead.
2. Floor in the double bottom.
3. Bracket between deck beam and side frame, together with bracket between side frame
and tank top plating, or margin plate.
4.The pillars in hole and tween deck.

Local Stress

These are created by such item:

1. Heavy concentrated load like boiler, engine etc.


2. Dead cargo such as timber
© All Rights Reserved.
Issue No. 1 Page 137 of 262
S E
MT
C
ST. THERESE- MTC COLLEGES SEAM 2
Iloilo, Philippines
E

Trim, Stability, Stress


R

C
E

OL
. T H

STUDENT LEARNING MODULE


LEGES
ST

Revision No. 2 Effectivity date: Reviewed by: Approved by:

01 September 2021 QMR President


3. Hull vibration
4. Ship resting on block on a dry dock (Static Stress)

Dynamic Forces

The dynamic effects arise from the motion of the ship itself. A ship among waves as three
linear motions.

1. Surging: The forward and aft linear motion (along x) of a ship is called surging.
2. Heaving: The vertical up and down linear motion (along y) of a ship is called heaving.
3. Swaying: The side to side linear motion (along z) of a ship is called swaying.
4. Rolling: The rotational motion of a ship about longitudinal axis is called rolling.
5. Yawing: The rotational motion of a ship about vertical axis is called yawing.
6. Pitching: The rotational motion of a ship about transverse axis is called pitching.

When the ship motions are large particularly in pitching and heaving, considerable
dynamic forces can be created in the structure.

Panting

 As wave passes along the ship, they cause fluctuation in water pressure which tends to
create in and out movement of the shell plating.
 This is particularly the case at the fore end.
 The rules of the classification societies required extra stiffening, at the end of the ship,
in the form of beams, brackets, stringer plate, etc. in order to reduce the possibility of
damage.
 This in and out movement is called panting.

© All Rights Reserved.


Issue No. 1 Page 138 of 262
S E
MT
C
ST. THERESE- MTC COLLEGES SEAM 2
Iloilo, Philippines
E

Trim, Stability, Stress


R

C
E

OL
. T H

STUDENT LEARNING MODULE


LEGES
ST

Revision No. 2 Effectivity date: Reviewed by: Approved by:

01 September 2021 QMR President

Slamming or Pounding

 In heavy weather when the ship is heaving and pitching, the fore end emerges from the
water and reenters with a slamming effect which is called pounding.
 Extra stiffening requires at the fore end to reduce the possibility of damage.

Racking
When a ship rolls in a seaway, it results in forces in the structure tending to distort it
transversely and may cause deformation at the corners. The deck tends to move laterally
relative to the bottom structure, and the shell on one side to move vertically relative to the
other side. This type of deformation is refered to as “racking”.

Uneven water pressure caused by wave action leads to distortion of the


structure as shown and is resisted by the Shear Stresses in the structure
including most significantly the Transverse bulkheads and framing. Racking
stresses are highest at the corners of the box section and thus the corner
brackets are specially inspected.

Calculates the pressure at any depth below the liquid surface

Hydrostatic pressure at any height below the water surface is calculated by P = hdg where
h is the height below the open water surface, d is the density of water and g is the
acceleration due to gravity. So, if you want to calculate gauge pressure at height h then use
formula P = hdg + P° where P° is atmospheric pressure.

Torsion
A ship traversing a wave train at an angle will be subject to a twisting moment
(torque) and the structure in „torsion‟. The greatest effect occurs with decks having large
openings. In some ships, a heavy torsion box girder including the upper deck is provided at

the topsides to accommodate the torsional stresses .

© All Rights Reserved.


Issue No. 1 Page 139 of 262
S E
MT
C
ST. THERESE- MTC COLLEGES SEAM 2
Iloilo, Philippines
E

Trim, Stability, Stress


R

C
E

OL
. T H

STUDENT LEARNING MODULE


LEGES
ST

Revision No. 2 Effectivity date: Reviewed by: Approved by:

01 September 2021 QMR President

Sloshing

slosh refers to the movement of liquid inside another object (which is, typically, also
undergoing motion). Sloshing or shifting cargo, water ballast, or other liquid (e.g. from leaks
or firefighting) can cause disastrous capsizing in ships due to free surface effect; this can
also affect trucks and aircraft.
The effect of slosh is used to limit the bounce of a roller hockey ball. Water slosh can
significantly reduce the rebound height of a ball but some amounts of liquid seem to lead to
a resonance effect. Many of the balls for roller hockey commonly available contain
water to reduce the bounce height.

© All Rights Reserved.


Issue No. 1 Page 140 of 262
S E
MT
C
ST. THERESE- MTC COLLEGES SEAM 2
Iloilo, Philippines
E

Trim, Stability, Stress


R

C
E

OL
. T H

STUDENT LEARNING MODULE


LEGES
ST

Revision No. 2 Effectivity date: Reviewed by: Approved by:

01 September 2021 QMR President

References

– 3G E-Learning. (2017). Essential seamanship. USA: 3G E-Learning LLC.


– 3G. E-Learning (2017). Ship and operation‟s management. USA: 3G E
Learning LLC
- Patterson, Chris J. & Jonathan D. Ridley. (2014). Ship stability, powering and
resistance. London: Adlard Coles Nautical.
- Arora, Shiven. (2014). Merchant ship naval architecture. New Delhi, India:
Venus books.
- Barrass, C.B. and Derrett, D.R. (2012). Ship stability for masters and mates. 7th
Ed. USA: Butterworth-Heinemann.

CHECKPOINT:

Activity 1
Identify

1. Due to the difference in the weight and buoyancy, which occur throughout the ship.
1. Cause by the motion of the ship at sea and the action of the wind and wave.
2. The straining of a ship lightly loaded amidships and more heavily forward and aft,
causing the bow and stern to be lower than the middle section.
3. The straining of a ship heavily loaded amidships and more lightly forward and aft,
causing the bow and stern to be higher than the middle section.
4. The rotational motion of a ship about transverse axis.
5. The vertical up and down linear motion (along y) of a ship.
6. The rotational motion of a ship about longitudinal axis.
7. Cause fluctuation in water pressure which tends to create in and out movement of
the shell plating.
8. Refers to the movement of liquid inside another object.
9. When a ship rolls in a seaway, it results in forces in the structure tending to distort it
transversely and may cause deformation at the corners.
© All Rights Reserved.
Issue No. 1 Page 141 of 262
S E
MT
C
ST. THERESE- MTC COLLEGES SEAM 2
Iloilo, Philippines
E

Trim, Stability, Stress


R

C
E

OL
. T H

STUDENT LEARNING MODULE


LEGES
ST

Revision No. 2 Effectivity date: Reviewed by: Approved by:

01 September 2021 QMR President

Identify the dynamic forces present in the picture.

Assessment

Enumerate:

1-3. Three ships stresses


4-5. Two types of longitudinal stress
6-9. Structure of ships that resists tranverse stress
10-13. Created local stress
14-19. Dynamic effects
20. Solution in calculating the pressure at any depth below the liquid surface

© All Rights Reserved.


Issue No. 1 Page 142 of 262
S E
MT
C
ST. THERESE- MTC COLLEGES SEAM 2
Iloilo, Philippines
E

Trim, Stability, Stress


R

C
E

OL
. T H

STUDENT LEARNING MODULE


LEGES
ST

Revision No. 2 Effectivity date: Reviewed by: Approved by:

01 September 2021 QMR President

Lesson 12 & 13

Learning Module 12.1 Draught, trim and stability (10 hours)

Competence, Course Outcomes, and Learning Outcomes

Competence:

A-II/1 F3.C2: Maintain seaworthiness of the ship

Course Outcome/s:

Calculate ship stability in compliance with the IMO intact stability


criteria under all conditions of loading

Learning Outcomes:

At the end of learning module, the student can:

.1. Calculate the draught change, trim due to loading or discharging a given weight
at a specified position.

Overview
There are only two types of trimming calculations. Other trimming calculations are
just variations of these two
fundamental types. In Case you
know the vessel's weight and center
of gravity (CG) location, and you seek
to find the forward and after
drafts. In Case II, you know the
forward and after drafts and desire to
know the vessel's weight and center
of gravity (CG) location. The simple
approach presented here should be
readily understandable by anyone
with a basic understanding of algebra
and geometry.

This article should benefit anyone involved with designing or operating any type of floating

© All Rights Reserved.


Issue No. 1 Page 143 of 262
S E
MT
C
ST. THERESE- MTC COLLEGES SEAM 2
Iloilo, Philippines
E

Trim, Stability, Stress


R

C
E

OL
. T H

STUDENT LEARNING MODULE


LEGES
ST

Revision No. 2 Effectivity date: Reviewed by: Approved by:

01 September 2021 QMR President


vessel. Regulatory agents and classification society employees also need to understand
these methods to check various designs for compliance. This article is also designed to give
a good theoretical understanding of the calculative methodology utilized in the trim and
stability sheets sold on this site.

Discussion

Key Concept: Stability

Ship stability is the ability of a ship to float in an upright position and, if inclined under the
action of an external force, to return to this position after the external force has ceased
acting.

Stability is not connected with a defined direction. However, ship inclination in the
transverse direction is the most common and easiest to achieve, and in practice, transverse
stability is the most critical to shipping safety.

A loaded ship's stability depends on its shape and dimensions, and its center of gravity's
actual location. Small ships with low freeboard are more prone to stability accidents than
other seagoing vessels.

After a ship is constructed, an operator does not influence the shape, dimensions, mass, and
location of an empty vessel's center of gravity. Still, he influences the final mass of the
loaded vessel and the center of gravity's location, defining the amount of cargo, stores, and
ballast water and their locations.

Several ship operations can adversely affect stability. Such effects must be understood and,
where possible, mitigated. When the liquid is consumed or removed from tanks, a free
surface is created, decreasing stability. When weight is lifted and suspended, its center of
gravity rises to the point of suspension. When a quantity of loose dry bulk cargo moves
transversely across the ship, it will list to one side with some stability loss. Phenomena such
as absorption of moisture by timber or similar deck cargoes, ice accretion on decks, and
shipped water accumulation will rise the vertical center of gravity VCG, reducing the righting
arm GZ.

Archimedes Principle
States that when a body is wholly or partially immersed in a fluid, it suffers a loss
in mass equal to the mass of the fluid it displaces
. Loss of weight is due to buoyancy
. When the mass of the ship changes, the mass of the water displaced changes by an equal
amount (floating body)
. For a body to float, it must displace its weight of water

© All Rights Reserved.


Issue No. 1 Page 144 of 262
S E
MT
C
ST. THERESE- MTC COLLEGES SEAM 2
Iloilo, Philippines
E

Trim, Stability, Stress


R

C
E

OL
. T H

STUDENT LEARNING MODULE


LEGES
ST

Revision No. 2 Effectivity date: Reviewed by: Approved by:

01 September 2021 QMR President


Displacement is the weight of water that a ship displaces when it is floating, which in turn
is the weight of a ship (and its contents).
Light displacement is the ship's weight excluding cargo, fuel, ballast, stores, passengers,
and crew, but with water in the boilers to steaming level.
Loaded displacement is the ship's weight, including cargo, passengers, fuel, water,
stores, dunnage, and other items necessary for use on a voyage, which brings the vessel
down to her load draft.
After perpendicular (AP) is the line perpendicular to the constructed waterline and the
rudder post
Freeboard- vertical distance for the waterline to the deck line
Gross tonnage is the vessel's internal volume less specific spaces such as peak and other
tanks for water ballast, particular light and air spaces, anchor gear, steering gear,
wheelhouse, galley, cabin for passengers, etc.
Net tonnage is the internal volume of spaces intended for cargo, revenue purposes

Trim using hydrostatic data:


It introduces the methods used for calculating trim in real situations. The level of accuracy
required using hydrostatic data. Consider the extract from the hydrostatic particulars for M/V
Almar.

© All Rights Reserved.


Issue No. 1 Page 145 of 262
S E
MT
C
ST. THERESE- MTC COLLEGES SEAM 2
Iloilo, Philippines
E

Trim, Stability, Stress


R

C
E

OL
. T H

STUDENT LEARNING MODULE


LEGES
ST

Revision No. 2 Effectivity date: Reviewed by: Approved by:

01 September 2021 QMR President

© All Rights Reserved.


Issue No. 1 Page 146 of 262
S E
MT
C
ST. THERESE- MTC COLLEGES SEAM 2
Iloilo, Philippines
E

Trim, Stability, Stress


R

C
E

OL
. T H

STUDENT LEARNING MODULE


LEGES
ST

Revision No. 2 Effectivity date: Reviewed by: Approved by:

01 September 2021 QMR President

For consistency, the accuracy used when interpolating for hydrostatic values shall be;
 Draught will be calculated to three decimal places, e.g., 2.345m.
 All other hydrostatic data values will be calculated to the same number of
decimal places

© All Rights Reserved.


Issue No. 1 Page 147 of 262
S E
MT
C
ST. THERESE- MTC COLLEGES SEAM 2
Iloilo, Philippines
E

Trim, Stability, Stress


R

C
E

OL
. T H

STUDENT LEARNING MODULE


LEGES
ST

Revision No. 2 Effectivity date: Reviewed by: Approved by:

01 September 2021 QMR President


 When calculating moments, values will be rounded to the nearest whole
number.

Proper Mean Draught when LCF is Amidships


A ship‟s hydrostatic particulars will only be accurate for the condition of trim
assumed. These hydrostatics have been developed with the vessel floating on an even keel.
If the LCF is amidships, the displacement given when entering the hydrostatic particulars
with the Arithmetic Mean Draught (AMD) will always be corrected.

AMD=

The waterline will rotate about F where the draught amidships, being the arithmetic mean
draught (AMD), will be the same as the draught at F, known as the True Mean Draught
(TMD).

When the LCF is amidships: TMD=AMD

20.1.2 True Mean Draught when LCF is not Amidships

Consideration of the hydrostatic particulars shows that the position of the LCF changes with
draught. The position of the LCF is seldom amidships.
If the weight is moved aft, the waterline will rotate about F, as shown. In this case, the
draught amidships, being AMD, reduces. Entering the hydrostatic particulars with this
reduced AMD will result in a lesser displacement value that will be an error.

© All Rights Reserved.


Issue No. 1 Page 148 of 262
S E
MT
C
ST. THERESE- MTC COLLEGES SEAM 2
Iloilo, Philippines
E

Trim, Stability, Stress


R

C
E

OL
. T H

STUDENT LEARNING MODULE


LEGES
ST

Revision No. 2 Effectivity date: Reviewed by: Approved by:

01 September 2021 QMR President

The error in the displacement will be equivalent to the weight of the slice or layer of water
shaded.

The difference between the displacement obtained for the Arithmetic Mean Draught and
obtained for the True Mean Draught is termed the layer correction. It will be additive or
subtractive from the Arithmetic Mean Draught displacement depending on whether the LCF
is aft or forward of amidships and whether the head or stern trims it.

20.1.3 Calculating the True mean Draught

Here a correction must be added to the AMD to obtain the TMD at the LCF.
Consider the similar triangles where:
=

𝑇𝑟𝑖𝑚 𝑥 𝐷𝑖𝑠𝑡𝑎𝑛𝑐𝑒 𝐿𝐶𝐹 𝑓𝑟𝑜𝑚 𝑎𝑚𝑖𝑑𝑠 𝑖𝑝


It follows that: Correction to AMD= 𝐿𝐵𝑃

Example 1
A ship LBP 148 m floats at draughts F 4.60 m A 5.80 m and has LCF 69.0 m foap. Calculate
the True Mean Draught.

Solution
Draw a simple sketch!
TMD= AMD + Correction
AMD= = 5. 20 m
Correction to AMD
= = 0.041 m

© All Rights Reserved.


Issue No. 1 Page 149 of 262
S E
MT
C
ST. THERESE- MTC COLLEGES SEAM 2
Iloilo, Philippines
E

Trim, Stability, Stress


R

C
E

OL
. T H

STUDENT LEARNING MODULE


LEGES
ST

Revision No. 2 Effectivity date: Reviewed by: Approved by:

01 September 2021 QMR President

Therefore; True Mean Draught = 5.200+0.041=5.241m (Ans)

In all cases, the correction to the AMD will be added or subtracted depending on two
factors:
1. Whether the ship is trimmed by the head or by the stern;
2. whether LCF is forward or aft of amidships.

Consider the four possibilities shown in the figure.

(1) Ship trimmed by stern, LCF (2) Ship trimmed by stern, LCF forward of
aft of amidships: amidships: TMD=AMD -Correction
TMD=AMD + correction

(3) Ship trimmed by head, LCF aft of (4) Ship trimmed by head, LCF forward of
amidships: TMD= AMD- Correction amidships: TMD= AMD + Correction

Example 2
A ship LBP 120 m floats at draughts F 4.62 m A 5.40 m. calculate the proper mean draught
if the LCF is 66 m foap.

Solution
TMD=AMD- Correction
AMD= = 5.01 m
Correction to AMD=
=0.039 m

© All Rights Reserved.


Issue No. 1 Page 150 of 262
S E
MT
C
ST. THERESE- MTC COLLEGES SEAM 2
Iloilo, Philippines
E

Trim, Stability, Stress


R

C
E

OL
. T H

STUDENT LEARNING MODULE


LEGES
ST

Revision No. 2 Effectivity date: Reviewed by: Approved by:

01 September 2021 QMR President

Therefore; True Mean Draught =5.010 – 0.039= 4.971 m (Ans)

Example 3
A ship LBP 166 m floats at draughts f 7.32 m A 6.84 m. Calculate the proper mean
draught if the LCF is 80 m foap.

Solution
TMD= AMD – Correction
AMD= = 7.08 m
Correction to AMD=
=0.009 m

Therefore; True Mean Draught= 7.080 – 0.009= 7.071 m (Ans)

The following formula may be used as an alternative provided that it is applied


precisely in the form in which it is stated below:

𝑑𝑎 𝑑𝐹
TMD=da – ( 𝑥𝐿𝐶𝐹 𝑓𝑜𝑎𝑝)
Where: da is the 𝐿𝐵𝑃 draught aft;
df is the draught forward;
© All Rights Reserved.
Issue No. 1 Page 151 of 262
S E
MT
C
ST. THERESE- MTC COLLEGES SEAM 2
Iloilo, Philippines
E

Trim, Stability, Stress


R

C
E

OL
. T H

STUDENT LEARNING MODULE


LEGES
ST

Revision No. 2 Effectivity date: Reviewed by: Approved by:

01 September 2021 QMR President


LCF foap is the position of the LCF expressed as being so many meters
forward of the aft perpendicular;
LBP is the length between perpendiculars.

The expression (da-dF) must be left as it is and not reversed. If the trim is by the
head, then (da- dF) will negatively value.

Example 4
A ship LBP 120 m floats at draughts F 4.62 m A 5.40 m. Calculate the proper mean
draught if the LCF is 66 m foap.

Solution
TMD= da- ( 𝑥 𝐿𝐶𝐹 𝑓𝑜𝑎𝑝)
TMD=5.40- ( 𝑥 66)
TMD=5.40- ( 𝑥 66)
TMD= 4.971 m (Ans)

Example 5
A ship LBP 166 m floats at draughts f 7.32 m A 6.84 m. Calculate the proper mean
draught if the LCF is 80 m foap.

Solution
TMD= da- ( 𝑥 𝐿𝐶𝐹 𝑓𝑜𝑎𝑝)
TMD=6.84- ( 𝑥 80)
TMD=6.84- ( 𝑥 80)
TMD=6.84 – (-0.231)
TMD=6.84 + 0.231
TMD=7.071 m (Ans)

20.2 Trim Calculations Using Hydrostatic Data-Taking Moments about the


Mean LCF

The first approach to solving trim problems is using Section 12- Introduction to Trim,
where moments are taken about the LCF.

The scenario to be considered is as follows. MV Almar arrives in port with particular


draughts forward and aft. Cargo is worked, being discharge, and loaded. The
anticipated draughts on completion of cargo must be calculated, and if necessary,
the ballast may have to be transferred to ensure a suitable trim for departure.

© All Rights Reserved.


Issue No. 1 Page 152 of 262
S E
MT
C
ST. THERESE- MTC COLLEGES SEAM 2
Iloilo, Philippines
E

Trim, Stability, Stress


R

C
E

OL
. T H

STUDENT LEARNING MODULE


LEGES
ST

Revision No. 2 Effectivity date: Reviewed by: Approved by:

01 September 2021 QMR President


The procedure for the calculation by taking moments about the mean LCF is as
follows:
1. Calculate the initial True Mean Draught (TMD).
2. Enter the hydrostatic particulars with the initial TMD and obtained values for
Displacement, MCTC, and LCF position, interpolating as necessary.
3. Calculate the final displacement.
4. Enter the hydrostatic particulars with final displacement value and obtained
values for TMD, MCTC, and LCF position, interpolating as necessary.
5. Calculate the mean MCTC and mean LCF position values.
6. Take moments about the mean LCF position to determine the net trimming
moments (by the head or the stern).
7. Using the mean MCTC value, calculate the change of trim (COT).
8. Apply the COT calculated in (7) to the initial trim and calculate the ship's final
trim.
9. Calculate the change of draughts aft and forward due to trim (Ta and Tf) by
apportioning the final trim by the position of the LCF for the final waterline.
10. Apply Ta and Tf to the final TMD obtained in (4) to obtain the final anticipated
draughts.
11. If necessary, calculate the change of trim required to bring the ship to the
desired departure trim, using the formula:
COT=
Calculate the weight of the ballast to be transferred between the chosen
tanks.

Example 6
MV Almar arrives in port with draughts F 4.00 m A 4.20 m.

Cargo is to be loaded as follows:


Hold No. 1 1400 t at lcg 144.94 m foap;
Hold No. 3 2900 t at lcg 103.94 m foap;
Hold No. 5 3104 t at lcg 62.05 m foap;
Hold No. 6 3285 t at lcg 42.31 m foap;

a. Calculate the final draughts.


b. Calculate the weight of ballast to transfer from 6 P + s DB ballast tank (lcg
42.19 m foap) to the forepeak tank (lcg 162.04 m foap) to ensure that the
ship sails with a trim of 0.60 m by the stern.
(LBP is 167.87 m for all calculations- see general particulars ion stability data book.)

Solution (a)
1. Calculate initial TMD AMD= = 4.10 m

Enter data with AMD and obtain approximate LCF position.


© All Rights Reserved.
Issue No. 1 Page 153 of 262
S E
MT
C
ST. THERESE- MTC COLLEGES SEAM 2
Iloilo, Philippines
E

Trim, Stability, Stress


R

C
E

OL
. T H

STUDENT LEARNING MODULE


LEGES
ST

Revision No. 2 Effectivity date: Reviewed by: Approved by:

01 September 2021 QMR President


AMD 4.10 m; LCF=86.78 m foap. Use this to calculate TMD.
TMD= da- ( 𝑥 𝐿𝐶𝐹 𝑓𝑜𝑎𝑝)
TMD= 4.20- ( 𝑥 86 78)
TMD 4.200- 0.103 =4.097 m

2. Enter data with TMD 4.097 m


Initial displacement= 11930 +(310 x )=122311
MCTC=308 t-m (no interpolar needed)
LCF=86.86 + (-0.08 x )= 86.78 m foap

3. Calculate the final displacement.


Initial displacement 12231
Load 1400
Load 2900
Load 3104
Load 3285
Final displacement 22920

4. Enter data with final displacement 22920 t

TMD=7.400 m; MCTC= 343 t-m; LCF= 85.10 m foap.


(N0 interpolation Needed)

5. Calculate mean MCTC and mean LCF


Mean MCTC= = 325.5 t-m. Use 326 t-m

Mean LCF= =85.985 m foap. Use 85.99 m foap


6. Take moments about the mean LCF position to determine net trimming
moments
Weight Mean Weigh Dist. Head Stern
(t) LCF (m lcg (m From moments Moments
foap) foap) mean (t-m) (t-m)
LCF (m)
1400 85.99 144.94 58.95 82530
2900 85.99 103.94 17.95 52055
3104 85.99 62.05 23.94 74310
3285 85.99 42.31 43.68 143489
Total 134585 217799
134585
83214

© All Rights Reserved.


Issue No. 1 Page 154 of 262
S E
MT
C
ST. THERESE- MTC COLLEGES SEAM 2
Iloilo, Philippines
E

Trim, Stability, Stress


R

C
E

OL
. T H

STUDENT LEARNING MODULE


LEGES
ST

Revision No. 2 Effectivity date: Reviewed by: Approved by:

01 September 2021 QMR President

7. Calculate the change of trim using the mean MCTC.


COT (cms)= = =255.3 cms.
COT= 2.553 m by the stern

8. Calculate the final trim.


Initial trim 0.200 m by the stern
COT (by the stern) 2.553 m
Final trim 2.753 m by the stern

9. Apportion the final trim using the LCF position for the final waterline.

TA= COT x Ta= 2.753x =1.397 m


Tf= COT- Ta Tf=2.753-1.397 = 1.356 m

10. Apply Ta and Tf to the final TMD to obtain final draughts.


fwd Aft
Final TMD 7.400 7.400
Trim (by stern) 1.356 1.397
Final draughts 6.044 8.797

Solution (b)
Trim on completion cargo 2.753 m by the stern
Required 0.600 m by the stern
COT required 2.153 m by the HEAD

COT (cms)= 215.3=


215= w= 616.2 tonnes to transfer (Ans)

20.3 Trim Calculations Using Hydrostatic Data- Trim by Consideration of


the Relative Position of the LCB and LCG

An alternative approach to trim problems is to consider the relative positions of the


LCB and LCG when the ship is momentarily „held‟ in the even keel condition. The
horizontal distance between the LCB and LCG represents a trimming lever that gives
a trimming moment when multiplied by the displacement. Knowledge of this and the
MCTC for the even keel waterline in question will allow the change of trim from the
even keel, the actual trim, to be determined.

© All Rights Reserved.


Issue No. 1 Page 155 of 262
S E
MT
C
ST. THERESE- MTC COLLEGES SEAM 2
Iloilo, Philippines
E

Trim, Stability, Stress


R

C
E

OL
. T H

STUDENT LEARNING MODULE


LEGES
ST

Revision No. 2 Effectivity date: Reviewed by: Approved by:

01 September 2021 QMR President

For the ship to float at even keel, the longitudinal positions of the center of gravity
(LCG) and the center of buoyancy (LCB) must be the same.

The weight is moved aft, causing a movement of G to G1 (LCG moves aft).


GG1=
Therefore: GG1 x W=w x d
Where: GG1 x W= Trimming moment= w x d

GG1 is the horizontal distance between the LCB and the LCG, representing a
trimming lever that will act to trim the ship by the stern.

Since: COT (cms)=


The change of the trim from the even keel may be calculated using:

𝑊 𝑥 𝐿𝐶𝐵~𝐿𝐶𝐺
COTFROM EVEN KEEL= 𝑀𝐶𝑇𝐶

Where: ‘W’ is the displacement of the ship


‘(LCB~LCG)’ is the horizontal distance between the LCB and the LCG that
exists when the ship is „held‟ at even keel
‘MCTC’ is the MCTC value for the current draught.
To calculate the position of the LCG for the final loaded condition, moments are
taken about the aft perpendicular in the same way as they would be taken about the
keel to find the final KG.

© All Rights Reserved.


Issue No. 1 Page 156 of 262
S E
MT
C
ST. THERESE- MTC COLLEGES SEAM 2
Iloilo, Philippines
E

Trim, Stability, Stress


R

C
E

OL
. T H

STUDENT LEARNING MODULE


LEGES
ST

Revision No. 2 Effectivity date: Reviewed by: Approved by:

01 September 2021 QMR President


20.3.1 Calculating the Final Draught- Ship Initially on an Even Keel

Example 7
MV Almar has an even keel draught of 5.10 m. cargo is worked as follows:

Discharge 356 t from lcg 148 m foap;


Discharge 404 t from lcg 59 m foap;
Load 566 t at lcg 120 m foap;
Load 800 t at lcg 102 m foap;
Load 200 t at lcg 81 m foap;
Load 1600 t at lcg 44 m foap.

Calculate the final draughts.


Solution
1. Calculate the initial TMD.
Because the ship is on an even keel: AMD=TMD= 5.100 m

2. Enter data with TMD and obtain Displacement and LCB values
Initial displacement is 15440 t
LCB is 87,09 m foap.
Because the ship is on an even keel, the positions of the LCB and the LCG will
be the same.
Therefore:
Initial LCG position= LCB position= 87.09 m foap

3. Take moments about the AP to find the final LCG position.


Weight (t) Weight (t) Lcg (m foap) Moments (t-m)
Initial displ. 15440 87.09 1344688
-356 148.00 -52688
-404 59.00 -23836
566 120.00 67920
800 102.00 81600
200 81.00 16200
1600 44.00 70400
Final 17846 84.29 1504266

4. Enter data with final displacement and obtain TMD, MCTC, LCB, and LCF
positions.
Final TMD = 5.80 + (0.1 x )= 5.847 m
MCTC= 319 y-m LCB=86.98 m foap
LCF=86.12 + (-0.03 x )= 86.11 m foap

© All Rights Reserved.


Issue No. 1 Page 157 of 262
S E
MT
C
ST. THERESE- MTC COLLEGES SEAM 2
Iloilo, Philippines
E

Trim, Stability, Stress


R

C
E

OL
. T H

STUDENT LEARNING MODULE


LEGES
ST

Revision No. 2 Effectivity date: Reviewed by: Approved by:

01 September 2021 QMR President

5. Calculate the change of trim from the even keel for the final condition.
~
COTFROM EVEN KEEL=
COTFROM EVEN KEEL= = 150.5 cms

A simple Sketch will indicate whether the COT is by the head or by the stern.

6. Apportion the change of trim from even keel to the forward and aft draughts,
applying Ta and Tf to the final even keel draught (TMD).
Ta=COT x Ta=150.5 x = 77.2 cms= 0.772 m
Tf=COT- Ta Tf=105.5- 77.2 = 73.3 cms= 0.733 m
Fwd Aft
Final TMD 5.847 5.847
Trim (by stern) 0.733 0.772
Final draughts 5.114 6.619

20.3.2 Calculating the Final Draught- Ship Initially Trimmed


If the ship is initially trimmed, then the LCB and LCG must be in different positions to
cause the ship to trim from the even keel condition. In this situation, the initial
proper mean draught must first be calculated, and the hydrostatic data entered to
determine initial values of displacement, MCTC, and the LCB position. The direction
to trim, whether by the head or stern, will be known.

Using the formula:

~
COTFROM EVEN KEEL=
Will allow the initial trimming lever causing the trim (LCB~LCG) to be calculated.
Knowledge of the direction of trim will allow the value of (LCB~LCG) to be applied to
the initial LCB to obtain an initial value of the position of LCG.
Example 8 (same as Example 6)
MV Almar arrives in port with draughts F 4.00 m A 4.20 m.
© All Rights Reserved.
Issue No. 1 Page 158 of 262
S E
MT
C
ST. THERESE- MTC COLLEGES SEAM 2
Iloilo, Philippines
E

Trim, Stability, Stress


R

C
E

OL
. T H

STUDENT LEARNING MODULE


LEGES
ST

Revision No. 2 Effectivity date: Reviewed by: Approved by:

01 September 2021 QMR President

Cargo is to be loaded as follows:


Hold No. 1 1400 t at lcg 144.94 m foap;
Hold No. 3 2900 t at lcg 103.94 m foap;
Hold No. 5 3104 t at lcg 62.05 m foap;
Hold No. 6 3285 t at lcg 42.31 m foap;

a. Calculate the final draughts.


b. Calculate the weight of ballast to transfer from 6 P + s DB ballast tank (lcg
42.19 m foap) to the forepeak tank (lcg 162.04 m foap) to ensure that the
ship sails with a trim of 0.60 m by the stern.

Solution (a)
1. Calculate initial TMD
AMD=4.10 m
Enter data with AMD and obtain approximate LCF position.
LCF=86.78 m foap
Use this to calculate TMD.
TMD= da- ( 𝑥 𝐿𝐶𝐹 𝑓𝑜𝑎𝑝)
TMD= 4.20- ( 𝑥 86 78)
TMD 4.200- 0.103 =4.097 m

2. Enter data with TMD 4.097 m


Initial displacement= 11930 +(310 x )=122311
MCTC=308 t-m LCB=87.27 m foap
3. Calculate the initial trimming lever (LCB~LCG)
~ ~
COTFROM EVEN KEEL= 20.0=
(LCB~LCG) = =0.50 m
4. Calculate initial LCG position.
The stern trims the ship. LCB is 87.27 m foap.
(LCB~LCG) is 0.50 m.

For the ship to be trimmed by the


stern, LCG must be aft of LCB,

© All Rights Reserved.


Issue No. 1 Page 159 of 262
S E
MT
C
ST. THERESE- MTC COLLEGES SEAM 2
Iloilo, Philippines
E

Trim, Stability, Stress


R

C
E

OL
. T H

STUDENT LEARNING MODULE


LEGES
ST

Revision No. 2 Effectivity date: Reviewed by: Approved by:

01 September 2021 QMR President


therefore:Initial LCG= LCB(LCB~LCG)

Initial LCG= 87.27- 0.50= 86.77 m foap


5. Take moments about the AP to find the final LCG.
Weight (t) Weight (t) Lcg (m foap) Moments (t-m)
Initial displ. 12231 86.77 1061284
1400 144.94 202916
2900 103.94 301426
3104 62.05 192603
3285 42.31 138988
Final 22920 82.78 1897217

6. Enter data with final displacement and obtain TMD, MCTC, LCB, and LCF
positions.

Final TMD= 7.400 m MCTC= 343 t-m


LCB= 86.70 m foap LCF= 85.19 m foap

7. Calculate the change of trim from even keel for the final condition.
~
COTFROM EVEN KEEL=
COTFROM EVEN KEEL= = 261.9 cms
A simple sketch will indicate whether the COT is by the head or by the stern.

8. Apportion the change of trim from even keel to the forward and aft draughts,
applying Ta and Tf to the final even keel draught (TMD).
Ta=COT x Ta=261.9x = 132.9 cms
Tf= COT- Ta Tf=261.9-132.9 = 129.0 cms

Fwd Aft
Final TMD 7.400 7.400
Trim (by stern) 1.290 1.329
Final draughts 6.110 8.729

Solution (b)
Trim on completion of Cargo 2.619 m by the stern
Required 0.600 m by the stern
© All Rights Reserved.
Issue No. 1 Page 160 of 262
S E
MT
C
ST. THERESE- MTC COLLEGES SEAM 2
Iloilo, Philippines
E

Trim, Stability, Stress


R

C
E

OL
. T H

STUDENT LEARNING MODULE


LEGES
ST

Revision No. 2 Effectivity date: Reviewed by: Approved by:

01 September 2021 QMR President


COT required 2.019 m by the HEAD

COT (cms)= 201.9=


201.9= w=577.8 tonnes to transfer (Ans)

Note: the answers obtained using the previous method where:


a. F 6.044 m A 8.797 m
b. 616.2 tonnes

Inaccuracy in these answers is caused by the assumption that the hydrostatic data
changed linearly between the range of draughts concerned when mean values of
MCTC and LCF are used to calculate trim change. Since no mean values are used in
the (LCB~LCG) method, the answers are much more accurate.

20.4 Various Examination Style Problems Involving Trim

20.4.1 Maximum Cargo to Load in Each Space for the Ship to Complete at
the Maximum Draught

Example 9
MV Almar is loading a bulk cargo in port and has draughts f 7.36 m A 8.24 m in
saltwater. Calculate the maximum amount of cargo to load in each of the spaces
available so that the maximum draught on departure will be 8.40 m. space is
available as follows:

No. 2 Hold, lcg 124.38 m foap;


No. 6 Hold, lcg 42.31 m foap.
(Note: the requirement is that the maximum amount of cargo is loaded, and the
draught is not to exceed 8.40 m. this means that the ship must complete on an even
keel draught of 8.40 m.)

Solution (a)- Taking moments about the mean LCF


1. Calculate initial TMD
AMD= 7.80 m
Enter data with AMD and obtain LCF position.
LCF = 84.82 m foap.
Use this to calculate the TMD.
TMD=dA- ⌈ 𝑥 𝐿𝐶𝐹 𝑓𝑜𝑎𝑝⌉
TMD= 8.24- * 𝑥 84 82+
TMD= 7.795 m

2. Enter data with TMD 7.795 m


© All Rights Reserved.
Issue No. 1 Page 161 of 262
S E
MT
C
ST. THERESE- MTC COLLEGES SEAM 2
Iloilo, Philippines
E

Trim, Stability, Stress


R

C
E

OL
. T H

STUDENT LEARNING MODULE


LEGES
ST

Revision No. 2 Effectivity date: Reviewed by: Approved by:

01 September 2021 QMR President

Initial Displacement = 23920 + (330 x )= 24234 t


MCTC= 351 t-m
LCF= 84.92 + (-0.1 x )= 84.83 m foap

Space Weight Mean Weight Dist. Head Stern


(t) LCF lcg From moments moments
(m foap) (m foap) LCF (m) (t-m) (t-m)

3. Enter data with required even keel draught (Final TMD) 8.40 m
Final displacement = 26270 t MCTC 365 t-m
LCF 84.17 m foap

4. Calculate cargo to load


Cargo to load= Final displacement – Initial displacement
Cargo to load= 26270- 24234= B2036 t

5. Calculate mean MCTC and mean LCF


Mean MCTC= = 358 t-m
Mean LCF= = 84.50 foap

6. Calculate COT required


Initial trim 0.88 m by the stern
Required trim 0.00 m even keel
COT required 0.88 m by the head

7. Calculate the required trimming moments to achieve this


COT= =
88=
Trimming moments= 31504 t-m by the head

8. Take moments about the mean LCF


Let x = cargo to load in No. 2 Hold; (2036 – x) cargo to load in No. 6 Hold

© All Rights Reserved.


Issue No. 1 Page 162 of 262
S E
MT
C
ST. THERESE- MTC COLLEGES SEAM 2
Iloilo, Philippines
E

Trim, Stability, Stress


R

C
E

OL
. T H

STUDENT LEARNING MODULE


LEGES
ST

Revision No. 2 Effectivity date: Reviewed by: Approved by:

01 September 2021 QMR President


No. 2 x 84.50 124.38 39.88 39.88x
No. 6 (2036-x) 84.50 42.31 42.19 (85899 –
42.19x)

Required head moments (31504 t-m) = Head moments – Stern moments


 31504 = 39.88x – (85899 – 42.19x)
 31504 = 39.88x – 85899 + 42.19x
 31504 + 85899 = 39.88x + 42.19x
 117403 = 82.07x
 x = 1431 tonnes in No. 2 Hold;
(2036 – 1431) = 605 tonnes in No. 6 Hold (Ans)

Solution (b) – By (LCB~LCG) method taking moments about AP


1. calculate initial TMD
AMD = 7.80 m
Enter data with AMD and obtain LCF position.
LCF = 84.82 m foap
Use this to calculate the TMD.

TMD = dA -( × 𝐿𝐶𝐹 𝑓𝑜𝑎𝑝)


TMD = 8.24 - ( × 84 82)
TMD = 7.795 m

2. Enter data with TMD 7.795 m


Initial Displacement = 23920 + (330 × ) = 24234 t
LCB = 86.61 m foap
MCTC = 351 t-m

3. Calculate the initial trimming lever (LCB~LCG)


Trim = 88 cms by the stern
× ~
COTFROM EVEN KEEL = 88 =
× ~

×
(LCB~LCG) = = 1.27 m

4. Calculate initial LCG position.


The stern trims the ship. LCB is 86.61 m foap.
(LCB~LCG) is 1.27 m.

© All Rights Reserved.


Issue No. 1 Page 163 of 262
S E
MT
C
ST. THERESE- MTC COLLEGES SEAM 2
Iloilo, Philippines
E

Trim, Stability, Stress


R

C
E

OL
. T H

STUDENT LEARNING MODULE


LEGES
ST

Revision No. 2 Effectivity date: Reviewed by: Approved by:

01 September 2021 QMR President

For a ship to be trimmed by the stern, LCG must be aft of LCB, therefore:
Initial LCG = LCB – (LCG~LCB)

Initial LCG = 86.61 – 1.27 = 85.34 m foap

5. Enter data will require even keel draught (Final TMD) 8.40 m
Final displacement = 26270 t MCTC 365 t-m
LCB = 86.44 m foap LCF 84.17 m foap

6. Calculate cargo to load


Cargo to load = Final displacement – Initial displacement
Cargo to load = 26270 – 24234 = 2036 t

7. Determine required final LCG


To complete on even keel LCG = LCB = 86.44 m foap

8. Take moments about the AP


Let x = cargo to load in No. 2 Hold; (2036 – x) cargo to load in No. 6 Hold

Weight (t) Icg (m foap) Moments (t-m)


Initial displ. 24234 85.34 2068130
No. 2 x 124.38 124.38x
No. 6 (2035 – x) 42.31 (86143 – 42.31x)
Final 26270 86.44

Note that the final displacement and final LCG are already known.
 86.44 =

 86.44 =
 2270779 = 2068130 + 124.38x + 86143 – 42.31x
 2270779 – 2068130 – 86143 = 124.38x - 42.31x
 116506 = 82.07x
x = 1420 tonnes in NO. 2 Hold
(2036 – 1420) = 616 tonnes in No. 2 Hold (Ans)

Note: again, it is seen that there is a difference in the answer obtained by the
different methods of calculation; 1431 tonnes in No.2 Hold and 605 tonnes in No. 6
Hold by the previous method, which is less accurate.

© All Rights Reserved.


Issue No. 1 Page 164 of 262
S E
MT
C
ST. THERESE- MTC COLLEGES SEAM 2
Iloilo, Philippines
E

Trim, Stability, Stress


R

C
E

OL
. T H

STUDENT LEARNING MODULE


LEGES
ST

Revision No. 2 Effectivity date: Reviewed by: Approved by:

01 September 2021 QMR President


20.4.2 Maximum Cargo to Load in each Space for the Ship to Complete at
the Load Displacement with the Desired Trim

Example 10
MV Almar has draughts F 9.46 m A 10.14 m in saltwater and completes loading to
the summer displacement. The remaining cargo is to be loaded into hold no. 2 and
holds no. 5. How much of the remaining cargo must be loaded into each hold for the
ship to complete cargo with a trim of 0.2 m by the stern.

Solution(a) – Taking moments about the mean LCF


1. Calculate initial TMD
AMD = 9.80 m
Enter data with AMD and obtain LCF position
LCF = 82.70 m foap
Use this to calculate with TMD.
TMD = dA - ( 𝐿𝐶𝐹 𝑓𝑜𝑎𝑝)
TMD = 10.14 ( × 82 70)
TMD = 9.805 m
2. Enter data with TMD 9.805 m
Initial Displacement = 31090 + (350 × ) = 31108 t
MCTC = 401 +(3 × )= 401 t -m
LCF = 82.70 +(−0 1 ) = 82.70 m foap
3. Calculate cargo to load
Cargo to load = Summer displacement – initial displacement
From ship particulars summer displacement = 32485 t
Cargo to load = 32485 – 31108 = 1377 t
4. Enter data with summer draught 10.20 m
MCTC = 411 t-m LCF = 82.32 m foap
5. Calculate mean MCTC and mean LCF
Mean MCTC = = 460 – m
Mean LCF = = 82.51 m foap
6. Calculate COT required
Initial trim 0.68 m by the stern
Required trim 0.20 m by the stern
COT required 0.48 m by the head
7. Calculate the required training moment to achieve this
COT = =
48 =
Trimming moments = 19488 t-m by the head
© All Rights Reserved.
Issue No. 1 Page 165 of 262
S E
MT
C
ST. THERESE- MTC COLLEGES SEAM 2
Iloilo, Philippines
E

Trim, Stability, Stress


R

C
E

OL
. T H

STUDENT LEARNING MODULE


LEGES
ST

Revision No. 2 Effectivity date: Reviewed by: Approved by:

01 September 2021 QMR President


8. Take moments about the mean LCF
Let x = cargo to load in no. 2 hold; (1377 – x) cargo to load in no. 5 hold
From stability data book: lcg no. 2 hold = 124.38 m foap;
lcg no. 5 hold = 62.05 m foap.
Space Weight Mean LCF Weight Dist from Head Stern
(t) (m foap) lcg (m mean moment moments
foap) LCF (m) s (t-m) (t-m)
N0. 2 x 82.51 124.38 41.87 41.87x
No. 5 (1377- 82.51 62.05 20.46 (28173 –
x) 20.46x)

Required head moments (19488 t-m =head moments – stern moments)


19488=41.87 x = (28173-20.46 x)
19488=41.87 x – 28173 + 2046x
19488 + 28173 = 4187x + 20.46 x
47661= 62.33 x
x= 765 tonnes in No. 2 Hold;
(1377-765= 612 tonnes in No. 5 Hold (Ans)

Solution (b) – by (LCB~LCG) Method taking Moment about AP


1. Calculate initial TMD
AMD= 9.80 m
Enter data with AMD and obtain LCF position
LCF= 82.70 m foap.
Use this to calculate the TMD
TMD= dA-* 𝑥 𝐿𝐶𝐹 𝑓𝑜𝑎𝑝+
TMD= 10.14-* 𝑥 82 70+
TMD=905 m

2. Enter data with TMD 9.805 m


Initial displacement=31090+(350 x )= 311080
MCTC=401+(3x )=401 t-m
LCD=85.97 + (-0.04 x )=85.97 m foap

3. Calculate the initial trimming lever (LCB~LCG)


Trim=68 cms by the stern
~ ~
COTFROM EVEN KEEL= 68=
(LCB~LCG) = =0.88 m

© All Rights Reserved.


Issue No. 1 Page 166 of 262
S E
MT
C
ST. THERESE- MTC COLLEGES SEAM 2
Iloilo, Philippines
E

Trim, Stability, Stress


R

C
E

OL
. T H

STUDENT LEARNING MODULE


LEGES
ST

Revision No. 2 Effectivity date: Reviewed by: Approved by:

01 September 2021 QMR President


4. Calculate initial LCG position
The ship trimmed by the stern. LCB is 85.97 m foap.
(LCB~LCG) is 0.88 m

For the ship to be trimmed by the stern, LCG


must be aft of LCB, therefore:
Initial LCG= LCB- (LCG~LCB)

Initial LCG= 85.97-0.88= 85.09 m foap


5. Calculate cargo to load
Cargo to load= Summer displacement=Initial displacement
From ship particulars, Summer displacement=32485 t
Cargo to load=32485- 31108= 1377 t
6. Enter data with Summer draught 10.20 m
MCTC=411 t-m LCB=85.82 m foap LCF=82.32 m foap
7. Calculate required final LCG to desired trim of 0.20 m by the stern
~ ~
COTFROM EVEN KEEL= 20=
(LCB~LCG) = = 0.25 m

For the ship trim by the stern, LCG must be aft of


the LCB, therefore:
LCG=LCB- (LCB~LCG)

Required final LCG=85.82-0.25=85.57 m foap

8. Take moments about AP


Let x= cargo to load in No. 2 Hold;(1377-x) cargo to load in No. 5 Hold
From stability data book: lcg no. 2 hold = 124.38 m foap;
lcg no. 5 hold = 62.05 m foap.
Weight (t) lcg (m foap) Moments (t-m)
Initial displ. 31108 85.09 2646980
No. 2 X 124.38 124.38x
No. 5 (1377 – x) 62.05 (85443-62.05x)
Final 32485 85.57

© All Rights Reserved.


Issue No. 1 Page 167 of 262
S E
MT
C
ST. THERESE- MTC COLLEGES SEAM 2
Iloilo, Philippines
E

Trim, Stability, Stress


R

C
E

OL
. T H

STUDENT LEARNING MODULE


LEGES
ST

Revision No. 2 Effectivity date: Reviewed by: Approved by:

01 September 2021 QMR President


Note that the final displacement and the final LCG are already known.
85.57=
85.57=
277941=2646980+124.38x+85443-62.05x
277941-2646980-85443=124.38x -62.05x
47318=62.33x
X=759 tonnes in No. 2 Hold;
(1377-759) =618 tonnes in No. 5 hold (Ans)

Note: Again, it is seen that there is a difference in the answers obtained by the
different methods of calculation; 765 tonnes in No. 2 Hold; 612 tonnes in No. 5 Hold
by taking moments about the mean LCF, which is less accurate.

Where to Load a Single Weight to Keep the aft Draught Constant

Example 11
MV Almar has draughts F 4.24 m A 4.98 m and 436 tonnes of deck cargo remains to
be loaded. Calculate:
a. The position foap to load the weight so that the aft draught remains constant;
b. The final draught forward.

Solution(a)

1. Calculate initial TMD


AMD=4.61 m
Enter data with AMD and obtain LCF position.
LCF=86.41+(-0.03x )=86.41 m foap
Use this to calculate the TMD.
TMD=dA-* 𝑥 𝐿𝐶𝐹 𝑓𝑜𝑎𝑝+
TMD=4.98-* 𝑥86 41+
TMD=4.599m
2. Enter data with TMD 4.599 m
Initial displacement=13520+(320+ )=13837 t
TPC=31.84 MCTC=310 t-m LCF=86.41 m foap
Note: with this type of problem, it is not usually required to use mean
hydrostatic values.
3. Consider Figure 20.16
If the 436 tonnes of cargo is initially loaded at the LCF, the vessel will
experience uniform Sinkage.
The cargo must then be shifted to a position forward of the LCF so that the
ship trims by the head to reduce the draught aft back to its original value.
© All Rights Reserved.
Issue No. 1 Page 168 of 262
S E
MT
C
ST. THERESE- MTC COLLEGES SEAM 2
Iloilo, Philippines
E

Trim, Stability, Stress


R

C
E

OL
. T H

STUDENT LEARNING MODULE


LEGES
ST

Revision No. 2 Effectivity date: Reviewed by: Approved by:

01 September 2021 QMR President


For the draught aft to remain constant:

Sinkage= Ta

Where Ta is the change of draught aft due to trim caused by moving the
weight forward.
Therefore: =COT x

Thus, to keep the draught aft constant when loading a known weight:
= x
Calculating the distance „d‟ will allow the position foap to be determined.

4. Calculate the position to load the weight


= x
= x
= 0.724d d=18.91 m forward of the LCF
Therefore, the weight must be loaded: 86.41 + 18.91 =105.32 m foap (Ans)
Solution(b)
The position to load the weight is now known.
Sinkage= Sinkage =13.7 cms
COT= COT= =26.6 cms
Ta= COT x Ta= 26.6x =13.7 cms
© All Rights Reserved.
Issue No. 1 Page 169 of 262
S E
MT
C
ST. THERESE- MTC COLLEGES SEAM 2
Iloilo, Philippines
E

Trim, Stability, Stress


R

C
E

OL
. T H

STUDENT LEARNING MODULE


LEGES
ST

Revision No. 2 Effectivity date: Reviewed by: Approved by:

01 September 2021 QMR President


Tf= COT- Ta Tf==26.6-13.7=12.9 cms

Calculate the final draughts.


Fwd Aft
Initial 4.240 4.980
Sinkage 0.137 0.137
4.377 5.117
Trim 0.129 0.137
Final draughts 4.506 4.980

Final forward draught= 4.506 m (Ans)


Calculating both draughts provides a part check (a) of the calculation.

Calculating the Weight to Load to Reduce the Deepest Draught by a Given


Amount
Example 12
MV Almar has draughts F 5.62 m A 6.48 m.
a. Calculate the ballast amount to pump into the forepeak tank (lcg 162.04 m
foap) to reduce the draught aft to 6.20 m.
b. Calculate the final draught forward.

Solution (a)
1. Calculate initial TMD
AMD=6.05 m
Enter with AMD and obtain an LCF position.
LCF=86.05+ (-0.04 x )= 86.03 m foap
Use this to calculate TMD.
TMD=dA -* 𝑥 𝐿𝐶𝐹 𝑓𝑜𝑎𝑝+
TMD= 6.48- * 𝑥86 03+
TMD=6.039 m
2. Enter data with TMD 6.039 m
Initial Displacement= 18340+(320 x )= 18465 t
TPC=32.34 + (0.04x )= 32.36
MCTC= 321 t-m
LCF=86.05+(-0.04x )= 86.03 m foap
Note: With this type of problem, it is not usually required to use mean
hydrostatic values.
3. If the weight (ballast) is initially loaded at the LCF, the vessel will experience
uniform sinkage.

© All Rights Reserved.


Issue No. 1 Page 170 of 262
S E
MT
C
ST. THERESE- MTC COLLEGES SEAM 2
Iloilo, Philippines
E

Trim, Stability, Stress


R

C
E

OL
. T H

STUDENT LEARNING MODULE


LEGES
ST

Revision No. 2 Effectivity date: Reviewed by: Approved by:

01 September 2021 QMR President


The weight must then be shifted to the forepeak tank so that the ship trims
by the head to give the required reduction in draught aft.
For the draught aft to be reduced:

Reduction in dA= Ta-Sinkage

Ta is the draught aft change due to trim caused by moving the weight (ballast)
forward.
Therefore: Reduction in dA=*𝐶𝑂𝑇 𝑥 +-

Thus, to reduce the draught aft: Reduction in dA=*


𝑤𝑥𝑑 𝑎 𝑤
𝑥 +-
𝑀𝐶𝑇𝐶 𝐿 𝑇𝑃𝐶

Calculating the „w‟ to determine the


weight of ballast to be loaded in the forepeak.
4. Calculate the reduction in draught aft required
Reduction in dA=6.48 – 6.20= 0.28 m =28 cms
5. Calculate the weight of ballast to load
Reduction in dA= * 𝑥 +-
28=* 𝑥 +-
28=0.121 w-0.031 w
28=0.090w w=311.1 tonnes (Ans)

Solution (b)
The weight of ballast water in the forepeak is now known.
Sinkage= Sinkage= =9.6 cms
COT= COT= =73.7 cms

Ta=COT x Ta=73.7x =37.8 cms


© All Rights Reserved.
Issue No. 1 Page 171 of 262
S E
MT
C
ST. THERESE- MTC COLLEGES SEAM 2
Iloilo, Philippines
E

Trim, Stability, Stress


R

C
E

OL
. T H

STUDENT LEARNING MODULE


LEGES
ST

Revision No. 2 Effectivity date: Reviewed by: Approved by:

01 September 2021 QMR President


Tf= COT-Ta Tf=73.7 – 37.8= 35.9 cms

Calculate the final draughts.


Fwd Aft
Initial 5.620 6.480
Sinkage 0.096 0.096
5.716 6.576
Trim 0.359 0.378
Final draughts 6.075 6.198

Final forward draught= 6.075 m (Ans)


Calculating both draughts provides a part check (a) of the calculation. The error of
0.002 m in the aft draught is due to the rounding up of figures in the calculation.

Change in draught and trim Due to Change in Water Density

Causes of a Change of Trim Due to Change of Water Density


The center of floatation is located at the geometric center of the waterplane area.
The center of buoyancy is at the geometric center of the ship's underwater volume.
Their longitudinal positions will rarely be the same. Consider the values of LCB and
LCF for MV Almar as plotted
against draught.

If LCB and LCF were to be in


the same longitudinal position
(as would be the case for a box-
shaped vessel on an even keel)
and the vessel was to move
from water of one density to
water of a different density, the
change in density would result
only in bodily sinkage or rise.
When LCF is not in the same
position as LCF, as is usually the
case, a ship will always
experience a trim change when
passing between water of different densities.
(a) Shows a ship on an even keel in saltwater at waterline W1 L1. In this
condition, the LCB is vertically below the LCG. On passing into freshwater,
being less dense, the ship will initially sink bodily to waterline W2 L2 (b).

© All Rights Reserved.


Issue No. 1 Page 172 of 262
S E
MT
C
ST. THERESE- MTC COLLEGES SEAM 2
Iloilo, Philippines
E

Trim, Stability, Stress


R

C
E

OL
. T H

STUDENT LEARNING MODULE


LEGES
ST

Revision No. 2 Effectivity date: Reviewed by: Approved by:

01 September 2021 QMR President

A. Ship in saltwater
B. On passing into freshwater ship experience bodily sinkage to waterline W1 L1

At the new waterline, the LCF (F1) position can be assumed to be the same as at the
initial waterline (F) since the draught will not have significantly changed.
The added layer of buoyancy due to sinkage will have its centroid at b, i.e., at the
same longitudinal positions as F and F1.

Because a volume of buoyancy has been effectively added, B will move directly
towards the centroid of buoyancy's added slice to B1. This creates a trimming couple
that, in this case, will cause the ship to trim by the head.

Calculating the Final Draughts


1. Calculate the bodily sinkage.
This will be the freshwater allowance for the displacement if the ship passes
from saltwater to freshwater or; the dock water allowance if the ship passes
from saltwater to dock water.

𝐷𝑖𝑠𝑝𝑙𝑎𝑐𝑒𝑚𝑒𝑛𝑡
fwa (mm)=
𝑥 𝑇𝑃𝐶𝑆𝑊

𝑅𝐷𝐷𝑊
dwa (mm)=FWA x

Fwa and dwa are so-termed to distinguish between the real freshwater allowance
(and dock water allowance) values that would apply to the ship when at the summer

© All Rights Reserved.


Issue No. 1 Page 173 of 262
S E
MT
C
ST. THERESE- MTC COLLEGES SEAM 2
Iloilo, Philippines
E

Trim, Stability, Stress


R

C
E

OL
. T H

STUDENT LEARNING MODULE


LEGES
ST

Revision No. 2 Effectivity date: Reviewed by: Approved by:

01 September 2021 QMR President


displacement. The displacement and TPC values are those corresponding to
saltwater for the proper mean draught in question.

2. Calculate the weight of the layer of buoyancy acquired at b using:

W (tonnes)= Sinkage (cms) x TPCLESSER DENSITY

The TPC value must be for the fresh (or less dense) water.
3. Calculate the trimming moment
Trimming moment (t-m) = w x horizontal distance B to b
Where: Bb=BF1 = (LCB~LCF)

Therefore: Trimming moment (t-m) = w x (LCB~LCF)

4. Calculate the change of trim due to density


𝑇𝑟𝑖𝑚𝑚𝑖𝑛𝑔 𝑚𝑜𝑚𝑒𝑛𝑡
COT (cms)= 𝑀𝐶𝑇𝐶2𝑛𝑑 𝐷𝐸𝑁𝑆𝐼𝑇𝑌

The MCTC must always be for the second-density since the ship only trims when at
the new waterline after counting for the sinkage.
5. Apportion the COT to find Ta and Tf according to the position of the LCF
6. Apply the sinkage, Ta, and Tf to the initial draughts to find the new water
density draughts.
Note: that steps (3) and (4) may be combined for ease of calculation.

Example 13
MV Almar has draughts F 9.28 m A 10.32 m when floating in saltwater.
Calculate the draughts in freshwater.

Solution
1. Calculate initial TMD
AMD= 9.80 m
Enter data with AMD and obtain LCF position.
LCF= 82.70 m foap
Use this to calculate the TMD.
TMD= dA- * 𝑥 𝐿𝐶𝐹 𝑓𝑜𝑎𝑝+
TMD= 10.32- * 𝑥 82 70+
TMD=9.808 m

2. Enter data with TMD 9.808 m


Displacement=31090 + (350 x ) = 31118 t

© All Rights Reserved.


Issue No. 1 Page 174 of 262
S E
MT
C
ST. THERESE- MTC COLLEGES SEAM 2
Iloilo, Philippines
E

Trim, Stability, Stress


R

C
E

OL
. T H

STUDENT LEARNING MODULE


LEGES
ST

Revision No. 2 Effectivity date: Reviewed by: Approved by:

01 September 2021 QMR President

TPC= 34.89 + (0.07 x )= 34.90


MCTC=401 + (3 x )= 401 t-m
LCB= 85.97 + (-0.04x )= 85.97 m foap
LCF= 82.70 +( -0.10 x )= 82.69 m foap
3. Calculate the bodily sinkage
fwa (mm)=
fwa (mm)= = 223 mm = 22.3 cms
4. Calculate the weight of the layer of buoyancy
W (tonnes)= Sinkage (cms) x TPCLESSER DENSITY WATER
W = 22.3 x 34.90 x =759.3 tonnes
5. Calculate the change of trim due to density
~
COT (cms)=
2

COT (cms)= = 6.4 cms


2
6. Apportion the COT to find Ta and Tf according to the position of the LCF
Ta= COT x Ta= 6.4 x =3.2 cms
Tf= COT-Ta Tf= 6.4- 3.2 = 3.2 cms
7. Apply the sinkage, Ta, and Tf to the initial draughts to find the new water
density draughts.
Fwd Aft
Initial 9.280 10.320
Sinkage 0.223 0.223
9.502 10.543
Trim 0.032 0.032
Final draughts 9.535 10.511
Note: in this example, the ship is trimmed by the head. Had the LCF been
forward of the LCB, the ship would have trimmed by the stern.

Alternative Formula for Calculating the Change of Trim


Having calculated the sinkage, the change of trim that arises due to change
of water density may be calculated using the following:
𝑊 𝑅𝐷 𝑅𝐷2 𝐿𝐶𝐹 𝐿𝐶𝐵
COT (cms)= 𝑅𝐷2 𝑥 𝑀𝐶𝑇𝐶2

Where: „W‟ is the ship displacement;


‘RD1’ is the density of the water from which the ship is leaving;
‘RD2’ is the density of water into which the ship is going;
‘MCTC2’ is the MCTC for the water into which the ship is going, and;

© All Rights Reserved.


Issue No. 1 Page 175 of 262
S E
MT
C
ST. THERESE- MTC COLLEGES SEAM 2
Iloilo, Philippines
E

Trim, Stability, Stress


R

C
E

OL
. T H

STUDENT LEARNING MODULE


LEGES
ST

Revision No. 2 Effectivity date: Reviewed by: Approved by:

01 September 2021 QMR President


LCF and LCB areas before. The term (LCF-LCB) must always be
calculated as it is and not changed around. Thus:
If the answer is positive, it is a change of trim by stern, and;
If the answer is negative, it is a change of trim by the head.

Considering Change of Trim Due to Change of Density when Conducting


Trim Problems

Example 14
MV Almar arrives in port with draughts F 8.60 m A 9.10 m in saltwater. The aft peak
tank is full. Calculate the amount of ballast water to discharge from the aft peak tank
so that the ship arrives at berth with trim by the stern of 0.20m. The RD of the dock
water at the berth is 1.006.

Solution
1. Calculate initial TMD
AMD=8.85 m
LCF=83.72 + (-0.1 x )= 83.67 m foap
Use this to calculate the TMD.
TMD= dA-* 𝑥 𝐿𝐶𝐹 𝑓𝑜𝑎𝑝+
TMD= 9.10− * 𝑥 83 67+
TMD= 8.851 m
2. Enter data with TMD 8.851 m
Displacement= 27630+(340 x )=27803 t
MCTC=375 + (3 x )= 377 t-m
LCB= 86.32 + (-0.03 x )= 86.30 m foap
LCF= 83.72 + (-0.10 x )= 83.67 m foap
3. Calculate the change of trim due to density when going from saltwater to
dock water RD 1.006
2
COT (cms)=
2

COT=
* +
2

COT= =- 3.7 cms


Because the answer is negative, the change of trim will be 3.7 cms by the head.
4. Calculate the change of trim required by discharging the ballast water
Initial trim 50.0 cms by the stern
Required 20.0 cms by the stern
COT required 30.0 cms by the head
COT (density) 03.7 cms by the head
COT (ballast discharge) 26.3 cms by the head
© All Rights Reserved.
Issue No. 1 Page 176 of 262
S E
MT
C
ST. THERESE- MTC COLLEGES SEAM 2
Iloilo, Philippines
E

Trim, Stability, Stress


R

C
E

OL
. T H

STUDENT LEARNING MODULE


LEGES
ST

Revision No. 2 Effectivity date: Reviewed by: Approved by:

01 September 2021 QMR President

Because the ship will trim by the head when it goes from saltwater to the
dock water, the change of trim required by discharging ballast from the aft
peak tank will be less. With this problem, consider that the ballast water is
discharged in the saltwater, causing a change of trim of 26.3 cms by the
head; the further 3.7 cms is achieved on passage from the saltwater to the
berth.

5. Calculate the amount of ballast to discharge from the aft peak tank (Lcg 4.66
m foap)
COT (cms)= 26.3=

W= 125.5 tonnes (Ans)

References

– 3G E-Learning . (2017) . Essential seamanship . USA : 3G E-Learning LLC.


– 3G . E-Learning (2017) . Ship and operation’s management . USA : 3G E
Learning LLC
- Patterson , Chris J. & Jonathan D. Ridley. (2014). Ship stability , powering and
resistance . London : Adlard Coles Nautical.
- Arora , Shiven. (2014). Merchant ship naval architecture . New Delhi , India :
Venus books.
- Barrass, C.B. and Derrett, D.R.(2012). Ship stability for masters and mates. 7 th
Ed. USA: Butterworth-Heinemann.

© All Rights Reserved.


Issue No. 1 Page 177 of 262
S E
MT
C
ST. THERESE- MTC COLLEGES SEAM 2
Iloilo, Philippines
E

Trim, Stability, Stress


R

C
E

OL
. T H

STUDENT LEARNING MODULE


LEGES
ST

Revision No. 2 Effectivity date: Reviewed by: Approved by:

01 September 2021 QMR President


CORRESPONDING COMPONENT FOR GRADES COMPUTATION:

A) ORAL COMPONENT :
Lesson 11” Action to be taken in the event of partial loss intact buoyancy.
Activity 1. Explain the action to be taken in the event of partial loss intact
buoyancy.
Instructions:
1) Students will explain activity 1 of lesson 11 as an Oral Presentation
correspond for Oral grades on Prelim Period.
2) Computation of grades will be based on the given Rubrics.

Rubrics

Criteria Poor Fair Satisfactory Very Good Excellent


1 2 3 4 5
1. Introduction

2. Comprehensive
Coverage of topic
3. Bearing
/Delivery
4. Deadline/Time

TOTAL = 20 pts
Transmutation Value
Pts. Grades (%)
1 = 50

20 = 9

B) RESEARCH COMPONENT:

Lesson 11. Stress Table & Stress calculating Equipments


Assessment: Enumeration
Instructions:
1) Students will conduct research work base on assessment question of lesson
11.
2) Submitted work will be graded based on Rubrics.
3) Accumulated grades will correspond to the Midterm Grades.

© All Rights Reserved.


Issue No. 1 Page 178 of 262
S E
MT
C
ST. THERESE- MTC COLLEGES SEAM 2
Iloilo, Philippines
E

Trim, Stability, Stress


R

C
E

OL
. T H

STUDENT LEARNING MODULE


LEGES
ST

Revision No. 2 Effectivity date: Reviewed by: Approved by:

01 September 2021 QMR President

CRITERIA POOR FAIR SATISFACTORY V.GOOD EXELLENT


1 2 3 4 5

1. Introduction
2. Comprehensive
Topics
3. Neatness
4. Grammar &
other Mechanics
5. Summary &
Deadline

TOTAL = 25 points
C) OUTPUT COMPONENT:
Lesson 9: Fresh water Allowance (FWA)
Activity 1&2: T/F , Multiple Choice
Instructions : Score will be transmuted equivalent to as Output grades on
prelim period.

Score :

1 POINT = 50 %

20 POINT = 98 %

© All Rights Reserved.

You might also like